Respiratory NCLEX

Lakukan tugas rumah & ujian kamu dengan baik sekarang menggunakan Quizwiz!

What should the nurse include when teaching the patient with COPD about the need for physical exercise? A. All patients with COPD should be able to increase walking gradually up to 20minutes per day. B. A bronchodilator inhaler should be used to relieve exercise-induced dyspnea immediately after exercise. C. Shortness of breath is expected during exercise but should return to baseline within 5 minutes after the exercise. D. Monitoring the heart rate before and after exercise is the best way to determine how much exercise can tolerated.

C Shortness of breath usually increases during exercise but the activity is not being overdone if breathing returns to baseline within 5 minutes after stopping. Bronchodilators can be administered 10 minutes before exercise but should not be administered for at least 5 minutes after activity to allow recovery. Patients are encouraged to walk 15 to 20 minutes per day with gradual increases but actual patterns will depend on patient tolerance. Dyspnea most frequently limits exercise and is a better indication of exercise tolerance tan is heart rate in the patient with COPD.

2. A client has a chest tube in place following a left lower lobectomy inserted after a stab wound to the chest. When repositioning the client, the nurse notices 200 cc of dark, red fluid flows into the collection chamber of the chest drain. What is the most appropriate nursing action? a. Clamp the chest tube b. Call the surgeon immediately c. Prepare for blood transfusion d. Continue to monitor the rate of drainage

Answer D. Blood that comes in contact with the pleural space becomes defibrinogenated and usually will not clot. It is not unusual for blood to collect in the chest and be released into the chest drain when the client changes position. The dark color of the blood indicates it is not fresh bleeding inside the chest

11. A client has a chest tube in place following a left lower lobectomy done after a stab wound to the chest. When repositioning the client, the nurse notices 200 cc of dark, red fluid flows into the collection chamber of the chest drain. What is the MOST appropriate nursing action? a. Clamp the chest tube b. Call the surgeon immediately c. Prepare for blood transfusion d. Continue to monitor the rate of drainage

Answer D. Blood that comes in contact with the pleural space becomes defibrinogenated and usually will not clot. It is not unusual for blood to collect in the chest and be released into the chest drain when the client changes position. The dark color of the blood indicates it is not fresh bleeding inside the chest.

The client with COPD may lose weight despite having adequate caloric intake. When counseling the client in ways to maintain an optimal weight, the nurse should tell the client to: A. Continue the same caloric intake and increase the amount of fat intake B. Increase his activity level to stimulate his appetite C. Increase the amount of complex carbohydrates and decrease the amount of fat intake D. Decrease the amount of complex carbohydrates while increasing calories, protein, vitamins, and minerals

Answer D. The client with COPD needs additional calories, protein, vitamins, and minerals. Answer A is incorrect because the client needs more calories but not more fat. Answer B is not feasible, will increase the O2 demands, and will result in further weight loss. Answer C leads to excess acid production and an increased respiratory workload.

For a client with asthma, the physician prescribes albuterol (Proventil), two puffs twice a day via MDI, and beclomethasone (Vanceril, Beclovent) two puffs twice a day via MDI. Which of the following instructions should the nurse provide the client? a) Administer medications 1 hr apart, 2 times a day. b) Administer albuterol first and follow with beclomethasone, 2 times a day. c) Administer albuterol on awakening, and alternate the medications every 4 hr. d) Administer beclomethasone inhaler first and follow with albuterol.

Answer: b The nurse instructs the client to administer the bronchodilator first (the beta-2 agonist always leads) in order to open the airway and allow for improved delivery of the corticosteroid to the lung tissue, which follows after 1 min between puffs. Using a spacer device with an MDI provides the best delivery of medication to the lungs.

Which of the following nursing interventions is most likely to prevent respiratory complications such as pneumonia and atelectasis in a postsurgical patient? a) Control of anxiety and agitation b) Adequate nutrition and fluids c) Adequate pain control d) Use of incentive spirometry

Answer: d Incentive spirometry improves lung expansion, helps expel anesthetic gases and mucus from the airway, and facilitates oxygenation of body tissues. Pain control and hydration may facilitate lung expansion and mobilization of secretions, but incentive spirometry directly increases lung volume and alveolar expansion.

In a man undergoing surgery, it was necessary to aspirate the contents of the upper gastro-intestinal tract. After surgery, the following values were obtained from an arterial blood sample: pH 7.55, PCO2 52 mm Hg and HCO3- 40 mmol/l. What is the underlying disorder? a) Metabolic acidosis b) Respiratory alkalosis c) Metabolic alkalosis d) Respiratory acidosis

C The pH is higher than normal, so there is an alkalosis. In addition, the PCO2 and bicarbonate concentration are higher than normal, so the patient is suffering from a metabolic alkalosis.

51. Which of the following statements made by a patient with COPD indicates a need for further education regarding the use of an ipratropium inhaler? A. "I should rinse my mouth following the two puffs to get rid of the bad taste." B. "I should wait at least 1 to 2 minutes between each puff of the inhaler." C. "If my breathing gets worse, I should keep taking extra puffs of the inhaler until I can breathe more easily." D. "Because this medication is not fast-acting, I cannot use it in an emergency if my breathing gets worse.

C. "If my breathing gets worse, I should keep taking extra puffs of the inhaler until I can breathe more easily." The patient should not take extra puffs of the inhaler at will to make breathing easier. Excessive treatment could trigger paradoxical bronchospasm, which would worsen the patient's respiratory status.

The healthcare provider auscultates the lungs of an adult patient. Soft, low pitched breath sounds are heard over the posterior lower lobes. Inspiration is longer than expiration. The healthcare provider interprets this assessment data as: Please choose from one of the following options. A. Mixed bronchial and bronchovesicular sounds B. Vesicular breath sounds normally heard in this location C. Bronchovesicular sounds usually heard D. Bronchial breath sounds normally heard in this location

B

The healthcare provider is providing teaching on pleural effusions. The healthcare provider understands that teaching has been effective when the patient states: Please choose from one of the following options. A. "A pleural effusion is accumulation of fluid in the airways of the lungs." B. "A pleural effusion is an accumulation of fluid in the pleural cavity." C. "A pleural effusion is an accumulation of blood in the airspace." D. "A pleural effusion is an accumulation of fluid in the alveoli."

B

pH 7.34, PaCO2 24, HCO3- 20 A. Respiratory Acidosis, Partially Compensated B. Metabolic Acidosis, Partially Compensated C. Metabolic Acidosis, Uncompensated D. Metabolic Alkalosis, Partially Compensated

B

pH 7.55, PaCO2 25, HCO3- 22 A. Respiratory Acidosis, Partially Compensated B. Respiratory Alkalosis, Uncompensated C. Metabolic Alkalosis, Partially Compensated D. Metabolic Acidosis, Uncompensated

B

pH 7.64, PaCO2 25, HCO3- 19 A. Respiratory Acidosis, Uncompensated B. Respiratory Alkalosis, Partially Compensated C. Respiratory Alkalosis, Uncompensated D. Metabolic Alkalosis, Partially Compensated

B

To validate the suspicion that a married male client has sleep apnea the nurse first: A. Asks the client if he experiences apnea in the middle of the night B. Questions the spouse if she is awakened by her husband's snoring C. Places the client on a continuous positive airway pressure (CPAP) device D. Schedules the client for a sleep tes

B Although this is a diagnostic tool, the first thing the nurse would do is question the spouse. This may lead to determining whether more tests are needed

43. The nurse evaluates that nursing interventions to promote airway clearance in a patient admitted with COPD are successful based on which of the following findings? A. Absence of dyspnea B. Improved mental status C. Effective and productive coughing D. PaO2 within normal range for the patient

C. Effective and productive coughing The issue of the question is airway clearance, which is most directly evaluated as successful if the patient can engage in effective and productive coughing.

A client is experiencing confusion and tremors is admitted to a nursing unit. An initial ABG report indicates that the PaCO2 level is 72 mm Hg, whereas the PaO2 level is 64 mm Hg. A nurse interprets that the client is most likely experiencing: A. Carbon monoxide poisoning B. Carbon dioxide narcosis C. Respiratory alkalosis D. Metabolic acidosis

B Carbon dioxide narcosis is a condition that results from extreme hypercapnia, with carbon dioxide levels in excess of 70 mm Hg. The client experiences symptoms such as confusion and tremors, which may progress to convulsions and possible coma.

When analgesics are ordered for a client with obstructive sleep apnea (OSA) following surgery, the nurse is most concerned about: A. Nonsteroidal antiinflammatory drugs (NSAIDs) B. Opioids C. Anticonvulsants D. Antidepressants E. Adjuvants

B Clients with obstructive sleep apnea are particularly sensitive to opioids. Thus the risk of respiratory depression is increased. The nurse must recognize that clients with OSA should start out receiving very low doses of opioids.

A company driver is found at the scene of an automobile accident in a state of emotional distress. He tells the paramedics that he feels dizzy, tingling in his fingertips, and does not remember what happened to his car. Respiratory rate is rapid at 34/minute. Which primary acid-base disturbance is the young man at risk for if medical attention is not provided? A. Respiratory Acidosis B. Respiratory Alkalosis C. Metabolic Acidosis D. Metabolic Alkalosis

B Hyperventilation is typically the underlying cause of respiratory alkalosis. Hyperventilation is also known as overbreathing. When someone is hyperventilating, they tend to breathe very deeply or very rapidly.

If a pleural effusion develops, which of the following actions best describes how the fluid can be removed from the pleural space and proper lung status restored? A. Inserting a chest tube B. Performing thoracentesis C. Performing paracentesis D. Allowing the pleural effusion to drain by itself

B Performing thoracentesis is used to remove excess pleural fluid. The fluid is then analyzed to determine if it's transudative or exudative. Transudates are substances that have passed through a membrane and usually occur in low protein states. Exudates are substances that have escaped from blood vessels. They contain an accumulation of cells and have a high specific gravity and a high lactate dehydrogenase level. Exudates usually occur in response to a malignancy, infection, or inflammatory process. A chest tube is rarely necessary because the amount of fluid typically isn't large enough to warrant such a measure. Pleural effusions can't drain by themselves.

George Kent is a 54 year old widower with a history of chronic obstructive pulmonary disease and was rushed to the emergency department with increasing shortness of breath, pyrexia, and a productive cough with yellow-green sputum. He has difficulty in communicating because of his inability to complete a sentence. One of his sons, Jacob, says he has been unwell for three days. Upon examination, crackles and wheezes can be heard in the lower lobes; he has a tachycardia and a bounding pulse. Measurement of arterial blood gas shows pH 7.3, PaCO2 68 mm Hg, HCO3 28 mmol/L, and PaO2 60 mm Hg. How would you interpret this? A. Respiratory Acidosis, Uncompensated B. Respiratory Acidosis, Partially Compensated C. Metabolic Alkalosis, Uncompensated D. Metabolic Acidosis, Partially, Compensated

B The patient has respiratory acidosis (raised carbon dioxide) resulting from an acute exacerbation of chronic obstructive pulmonary disease, with partial compensation.

Three-year-old Adrian is admitted to the hospital with a diagnosis of asthma and respiratory distress syndrome. The mother of the child reports to the nurse on duty that she has witnessed slight tremors and behavioral changes in her child over the past four days. The attending physician orders routine ABGs following an assessment of the ABCs. The ABG results are pH 7.35, PaCO2 72 mmHg and HCO3 38 mEq/L. What acid-base disorder is shown? A. Respiratory Acidosis, Uncompensated B. Respiratory Acidosis, Fully Compensated C. Respiratory Alkalosis, Fully Compensated D. Metabolic Alkalosis, Partially Compensated

B The patient has respiratory acidosis (raised carbon dioxide) resulting from asthma and respiratory distress syndrome, with compensation having normal pH value within 7.35to 7.45, increased PaCO2 which is acidic and increased HCO3 which is basic.

Mr. Wales, who underwent post-abdominal surgery, has a nasogastric tube. The nurse on duty notes that the nasogastric tube (NGT) is draining a large amount (900 cc in 2 hours) of coffee ground secretions. The client is not oriented to person, place, or time. The nurse contacts the attending physician and STAT ABGs are ordered. The results from the ABGs show pH 7.57, PaCO2 37 mmHg and HCO3 30 mEq/L. What is your assessment? A. Metabolic Acidosis, Uncompensated B. Metabolic Alkalosis, Uncompensated C. Respiratory Alkalosis, Uncompensated D. Metabolic Alkalosis, Partially Compensated

B The postoperative client's ABG results show that he has metabolic alkalosis because of an increased pH and HCO3. It is uncompensated due to the normal PaCO2 which is within 35 to 45 mmHg.

23. Which of the following positions is most appropriate for the nurse to place a patient experiencing an asthma exacerbation? A. Supine B. Lithotomy C. High-Fowler's D. Reverse Trendelenburg

C. High-Fowler'sThe patient experiencing an asthma attack should be placed in high-Fowler's position to allow for optimal chest expansion and enlist the aid of gravity during inspiration.

Mrs. Johansson, who had undergone surgery in the post-anesthesia care unit (PACU), is difficult to arouse two hours following surgery. Nurse Florence in the PACU has been administering Morphine Sulfate intravenously to the client for complaints of post-surgical pain. The client's respiratory rate is 7 per minute and demonstrates shallow breathing. The patient does not respond to any stimuli! The nurse assesses the ABCs (remember Airway, Breathing, Circulation!) and obtains ABGs STAT! Measurement of arterial blood gas shows pH 7.10, PaCO2 70 mm Hg and HCO3 24 mEq/L. What does this mean? A. Respiratory Alkalosis, Partially Compensated B. Respiratory Acidosis, Uncompensated C. Metabolic Alkalosis, Partially Compensated D. Metabolic Acidosis, Uncompensated

B The results show that Mrs. Johansson has respiratory acidosis because of decreased pH and increased PaCO2 which mean acidic in nature. Meanwhile, it is uncompensated because HCO3 is within the normal range.

A client has active TB. Which of the following symptoms will he exhibit? A. Chest and lower back pain B. Chills, fever, night sweats, and hemoptysis C. Fever of more than 104*F and nausea D. Headache and photophobia

B Typical signs and symptoms are chills, fever, night sweats, and hemoptysis. Chest pain may be present from coughing, but isn't usual. Clients with TB typically have low-grade fevers, not higher than 102*F. Nausea, headache, and photophobia aren't usual TB symptoms.

For a client diagnosed with epistaxis, which intervention would be included in the care plan? A. Performing several abdominal thrust (Heimlich) maneuvers B. Compressing the nares to the septum for 5 to 10 minutes C. Applying an ice collar to the neck area D. Encouraging warm saline throat gargles

B When a client experiences epistaxis, the nurse should compress the soft outer portion of the nares against the septum for approximately 5 to 10 minutes. the client should sit upright, breathe through the mouth, and refrain from talking. Performing abdominal thrusts is appropriate for the client with a foreign-body aspiration. Applying an ice collar to the neck is commonly done for a client after a tonsillectomy. Warm saline throat gargles are appropriate for the client with pharyngitis.

22. A client who experiences repeated pleural effusions from inoperable lung cancer is to undergo pleurodesis. The nurse plans to assist with which of the following after the physician injects the sclerosing agent through the chest tube? a) ambulate the client b) clamp the chest tube c) ask the client to cough and deep breathe d) ask the client to remain in one position only

B - After injection of the sclerosing agent, the chest tube is clamped to prevent the agent from draining back out of the pleural space. A repositioning schedule is used by some physicians, but its usefulness in dispersing the substance is controversial. Ambulation, coughing, and deep breathing have no specific purpose in the immediate period after injection.

A client is admitted to the hospital with a diagnosis of right lower lobe pneumonia. The nurse auscultates the right lower lobe, expecting to note which of the following types of breath sounds? a) absent b) bronchial c) vesicular d) bronchovesical

B - Bronchial sounds are normally heard over the trachea. The client with pneumonia will have bronchial breath sounds over area(s) of consolidation, because the consolidated tissue carries bronchial sounds to the peripheral lung fields. The client may also have crackles in the affected area resulting from fluid in the interstitium and alveoli. Absent breath sounds are not likely to occur unless a serious complication of the pneumonia occurs. Bronchovesicular sounds are normally heard over the main bronchi. Vesicular sounds are normally heard over the lesser bronchi, bronchioles, and lobes.

A client with bacterial pneumonia is to be started on I.V. antibiotics. Which of the following diagnostic tests must be completed before antibiotic therapy begins? A. Urinalysis. B. Sputum culture. C. Chest radiograph. D. Red blood cell count.

B A sputum specimen is obtained for culture to determine the causative organism. After the organism is identified, an appropriate antibiotic can be prescribed. Beginning antibiotic therapy before obtaining the sputum specimen may alter the results of the test. Neither a urinalysis, a chest radiograph, nor a red blood cell count needs to be obtained before initiation of antibiotic therapy for pneumonia.

Presence of overdistended and non-functional alveoli is a condition called: A. BrOnchitis B. Emphysema C. Empyema D. Atelectasis

B An overdistended and non-functional alveoli is a condition called emphysema. Atelectasis is the collapse of a part or the whole lung. Empyema is the presence of pus in the lung.

1. When caring for a client with a post right thoracotomy who has undergone an upper lobectomy, the nurse focuses on pain management to promote: a. Relaxation and sleep b. Deep breathing and coughing c. Incisional healing d. Range of motion exercises

B Answer B. The priority is postoperative respiratory toilet. This client will quickly develop profound atelectasis and eventually pneumonia without adequate gas exchange. This will only be achieved with the appropriate pain management.

The healthcare provider is caring for a patient who has a pneumothorax. When assessing the patient and the chest tube drainage system, a large fibrin clot is noted in the tubing. Which additional assessment finding requires immediate action by the healthcare provider? Please choose from one of the following options. A. Fluctuations in the water seal chamber B. A downward trend in blood pressure C. Increasing pain at the insertion site D. Decreased water in the suction control chamber

B Clots in the system can cause occlusion and lead to a tension pneumothorax, which may be evidenced by a downward trend in blood pressure as increased pressure on the heart and great vessels impair cardiac output.

Which dietary modification helps to meet the nutritional needs of patients with COPD? A. Eating a high-carbohydrate, low-fat diet B. Avoiding foods that require a lot of chewing C. Preparing most foods of the diet to be eaten hot D. Drinking fluids with meals to promote digestion

B Eating is an effort for patients with COPD and frequently these patients do not eat because of fatigue, dyspnea, and difficulty holding their breath while swallowing. Foods that require much chewing cause more exhaustion and should be avoided. A low carbohydrate diet is indicated if the patient has hypercapnia because carbohydrates are metabolized into carbon dioxide. Fluids should be avoided at meals to prevent a full stomach and cold foods seem to give less of a sense of fullness than hot foods.

All of the following nursing diagnoses are important for a client with chronic pulmonary emphysema (COPD). Which would receive priority when planning nursing interventions? A. Ineffective airway clearance B. Impaired gas exchange C. Self-care deficit D. Activity Intolerance

B Physical assessment of older persons with COPD might find ineffective airway clearance, but the classic physiology alteration is the destruction of the alveoli with resulting complications. Therefore, impaired gas exchange always is present and is the PRIORITY nursing intervention.

When the nurse is caring for an obese patient with left lower-lobe pneumonia, gas exchange will be best when the patient is positioned a. on the left side. b. on the right side. c. in the high-Fowler's position. d. in the tripod position.

B Rationale: The patient should be positioned with the "good" lung in the dependent position to improve the match between ventilation and perfusion. The obese patient's abdomen will limit respiratory excursion when sitting in the high-Fowler's or tripod positions.

A client comes to the emergency department with status asthmaticus. His respiratory rate is 48 breaths/minute, and he is wheezing. An arterial blood gas analysis reveals a pH of 7.52, a partial pressure of arterial carbon dioxide (PaCO2) of 30 mm Hg, PaO2 of 70 mm Hg, and bicarbonate (HCO3??') of 26 mEq/L. What disorder is indicated by these findings? a. Metabolic acidosis b. Respiratory alkalosis c. Metabolic alkalosis d. Respiratory acidosis

B Respiratory alkalosis results from alveolar hyperventilation. It's marked by a decrease in PaCO2 to less than 35 mm Hg and an increase in blood pH over 7.45. Metabolic acidosis is marked by a decrease in HCO3? to less than 22 mEq/L, and a decrease in blood pH to less than 7.35. In respiratory acidosis, the pH is less than 7.35 and the PaCO2 is greater than 45 mm Hg. In metabolic alkalosis, the HCO3? is greater than 26 mEq/L and the pH is greater than 7.45.

A client's arterial blood gas values are as follows: pH, 7.31; PaO2, 80 mm Hg; PaCO2, 65 mm Hg; HCO3 −, 36 mEq/ L. The nurse should assess the client for? A. Cyanosis. B. Flushed skin C. Irritability. D. Anxiety.

B The high PaCO2 level causes flushing due to vasodilation. The client also becomes drowsy and lethargic because carbon dioxide has a depressant effect on the central nervous system. Cyanosis is a sign of hypoxia. Irritability and anxiety are not common with a PaCO2 level of 65 mm Hg but are associated with hypoxia.

A group of nursing students are studying for a test over acid-base imbalance. One student asks another what the major chemical regulator of plasma pH is. What should the second student respond? a. Renin-angiotensin-aldosterone system b. Bicarbonate-carbonic acid buffer system c. Sodium-potassium pump d. ADH-ANP buffer system

B The major chemical regulator of plasma pH is the bicarbonate-carbonic acid buffer system. Therefore options A and C are incorrect. Option D does not exist, it is only a distractor for this question.

An oxygen delivery system is prescribed for a patient with COPD to deliver precise oxygen concentration. Which oxygen delivery system would the nurse anticipate to be prescribed? A. Face tent B. Venturi mask C. Aerosol mask D. Tracheostomy collar

B The venturi mask delivers the most accurate oxygen concentration. It is the best option for the client with chronic airflow limitation because it delivers a precise oxygen concentration.

A client has the following arterial blood gas (ABG) values: pH, 7.12; partial pressure of arterial carbon dioxide (PaCO2), 40 mm Hg; and bicarbonate (HCO3-), 15 mEq/L. These ABG values suggest which disorder? a. Metabolic alkalosis b. Metabolic acidosis c. Respiratory acidosis d. Respiratory alkalosis

B This client's pH value is below normal, indicating acidosis. The HCO3- value also is below normal, reflecting an overwhelming accumulation of acids or excessive loss of base, which suggests metabolic acidosis. The PaCO2 value is normal, indicating absence of respiratory compensation. These ABG values eliminate respiratory alkalosis, respiratory acidosis, and metabolic alkalosis.

45. The nurse reviews pursed lip breathing with a patient newly diagnosed with emphysema. The nurse reinforces that this technique will assist respiration by which of the following mechanisms? A. Preventing bronchial collapse and air trapping in the lungs during exhalation B. Increasing the respiratory rate and giving the patient control of respiratory patterns C. Loosening secretions so that they may be coughed up more easily D. Promoting maximal inhalation for better oxygenation of the lungs

45. The nurse reviews pursed lip breathing with a patient newly diagnosed with emphysema. The nurse reinforces that this technique will assist respiration by which of the following mechanisms? A. Preventing bronchial collapse and air trapping in the lungs during exhalation B. Increasing the respiratory rate and giving the patient control of respiratory patterns C. Loosening secretions so that they may be coughed up more easily D. Promoting maximal inhalation for better oxygenation of the lungs

8. A 58-year-old client with a 40-year history of smoking one to two packs of cigarettes a day has a chronic cough producing thick sputum, peripheral edema, and cyanotic nail beds. Based on this information, he most likely has which of the following conditions? a. Adult respiratory distress syndrome (ARDS) b. Asthma c. Chronic obstructive bronchitis d. Emphysema

8. ANSWER C. Because of his extensive smoking history and symptoms, the client most likely has chronic obstructive bronchitis. Clients with ARDS have acute symptoms of and typically need large amounts of oxygen. Clients with asthma and emphysema tend not to have a chronic cough or peripheral edema.

In order to prevent ventilator associated/ acquired pneumonia (VAP) in an intubated patient, the healthcare provider should: Please choose from one of the following options. A. Provide oral care and suctioning to the patient B. Maintain intubation for no more than 72727272 hours C. Give prophylactic antibiotics D. Discourage patient visitors

A

pH 7.45, PaCO2 50, HCO3- 30 A. Metabolic Alkalosis, Fully Compensated B. Respiratory Alkalosis, Fully Compensated C. Metabolic Alkalosis, Partially Compensated D. Respiratory Acidosis, Partially Compensated

A

A 24-year-old client comes into the clinic complaining of right-sided chest pain and shortness of breath. He reports that it started suddenly. The assessment should include which of the following interventions? A. Auscultation of breath sounds B. Chest x-ray C. Echocardiogram D. Electrocardiogram (ECG)

A Because the client is short of breath, listening to breath sounds is a good idea. He may need a chest x-ray and an ECG, but a physician must order these tests. Unless a cardiac source for the client's pain is identified, he won't need an echocardiogram.

The public health nurse is providing follow-up care to a client with TB who does not regularly take his medication. Which nursing action would be most appropriate for this client? A. Ask the client's spouse to supervise the daily administration of the medications. B. Visit the clinic weekly to ask him whether he is taking his medications regularly. C. Notify the physician of the client's non-compliance and request a different prescription. D. Remind the client that TB can be fatal if not taken properly

A Directly observed therapy (DOT) can be implemented with clients who are not compliant with drug therapy. In DOT, a responsible person, who may be a family member or a health care provider, observes the client taking the medication. Visiting the client, changing the prescription, or threatening the client will not ensure compliance if the client will not or cannot follow the prescribed treatment.

An elderly client with pneumonia may appear with which of the following symptoms first? A. Altered mental status and dehydration B. Fever and chills C. Hemoptysis and dyspnea D. Pleuritic chest pain and cough

A Fever, chills, hemoptysis, dyspnea, cough, and pleuritic chest pain are common symptoms of pneumonia, but elderly clients may first appear with only an altered mental status and dehydration due to a blunted immune response.

A comatose client needs a nasopharyngeal airway for suctioning. After the airway is inserted, he gags and coughs. Which action should the nurse take? A. Remove the airway and insert a shorter one B. Reposition the airway C. Leave the airway in place until the client gets used to it. D. Remove the airway and attempt suctioning without it

A If a client gags or coughs after nasopharyngeal airway placement, the tube may be too long. The nurse should remove it and insert a shorter one. Simply repositioning the airway won't solve the problem. The client won't get used to the tube because it's the wrong size. Suctioning without a nasopharyngeal airway causes trauma to the natural airway.

Which of the following medications should the nurse anticipate the physician would prescribe for "rescue" from sudden-onset wheezing, tightness in the chest, or shortness of breath due to an asthma diagnosis? A. albuterol inhaler (Proventil, Ventolin) B. salmeterol (Servent) C. formoterol (Foradil) D. cromolyn inhaler (Intal)

A Long acting beta2-agonists are not prescribed as rescue medications because of their delayed onset and longer duration.

Which of the following symptoms is common in clients with TB? A. Weight loss B. Increased appetite C. Dyspnea on exertion D. Mental status changes

A TB typically produces anorexia and weight loss. Other signs and symptoms may include fatigue, low-grade fever, and night sweats.

Decreased plasma bicarbonate (HCO3-) A. Metabolic Acidosis B. Respiratory Alkalosis C. Metabolic Alkalosis D. Respiratory Acidosis

A The body compensates by using body fat for energy, producing abnormal amounts of ketone bodies. In an effort to neutralize the ketones and maintain the acid-base balance of the body, plasma bicarbonate is exhausted. This condition can develop in anyone who does not eat an adequate diet and whose body fat must be burned for energy. Symptoms include headache and mental dullness.

What two organs in the body serve as a compensatory function to maintain acid base balance? A. Kidneys and Lungs B. Lungs and Spleen C. Heart and Liver D. Gallbladder and Appendix

A The carbonic acid concentration is controlled by the amount of carbon dioxide excreted by the lungs. The bicarbonate concentration is controlled by the kidneys, which selectively retain or excrete bicarbonate in response to the body's needs.

A client with a productive cough, chills, and night sweats is suspected of having active TB. The physician should take which of the following actions? A. Admit him to the hospital in respiratory isolation B. Prescribe isoniazid and tell him to go home and rest C. Give a tuberculin test and tell him to come back in 48 hours and have it read D. Give a prescription for isoniazid, 300 mg daily for 2 weeks, and send him home

A The client is showing s/s of active TB and, because of the productive cough, is highly contagious. He should be admitted to the hospital, placed in respiratory isolation, and three sputum cultures should be obtained to confirm the diagnosis. He would most likely be given isoniazid and two or three other antitubercular antibiotics until the diagnosis is confirmed, then isolation and treatment would continue if the cultures were positive for TB. After 7 to 10 days, three more consecutive sputum cultures will be obtained. If they're negative, he would be considered non-contagious and may be sent home, although he'll continue to take the antitubercular drugs for 9 to 12 months.

A nurse is teaching a client with TB about dietary elements that should be increased in the diet. The nurse suggests that the client increase intake of: A. Meats and citrus fruits B. Grains and broccoli C. Eggs and spinach D. Potatoes and fish

A The nurse teaches the client with TB to increase intake of protein, iron, and vitamin C.

A cigarette vendor was brought to the emergency department of a hospital after she fell into the ground and hurt her left leg. She is noted to be tachycardic and tachypneic. Painkillers were carried out to lessen her pain. Suddenly, she started complaining that she is still in pain and now experiencing muscle cramps, tingling, and paraesthesia. Measurement of arterial blood gas reveals pH 7.6, PaO2 120 mm Hg, PaCO2 31 mm Hg, and HCO3 25 mmol/L. What does this mean? A. Respiratory Alkalosis, Uncompensated B. Respiratory Acidosis, Partially Compensated C. Metabolic Alkalosis, Uncompensated D. Metabolic Alkalosis, Partially Compensated

A The primary disorder is acute respiratory alkalosis (low CO2) due to the pain and anxiety causing her to hyperventilate. There has not been time for metabolic compensation.

20. A client with pleural effusion has a thoracentesis, and a sample of fluid was sent to the laboratory. Analysis of the fluid reveals a high red blood cell count. The nurse interprets that this result is most consistent with: a) trauma b) infection c) heart failure d) liver failure

A - Pleural effusion that has a high red blood cell count may result from trauma and may be treated with placement of a chest tube for drainage. Other causes of pleural effusion include infection, heart failure, liver or renal failure, malignancy, or inflammatory processes. Infection would be accompanied by white blood cells. The fluid portion of the serum would accumulate with liver failure and heart failure.

When developing a discharge plan to manage the care of a client with chronic obstructive pulmonary disease (COPD), the nurse should advise the the client to expect to: A. Develop respiratory infections easily. B. Maintain current status. C. Require less supplemental oxygen. D. Show permanent improvement.

A A client with COPD is at high risk for development of respiratory infections. COPD is slowly progressive; therefore, maintaining current status and establishing a goal that the client will require less supplemental oxygen are unrealistic expectations. Treatment may slow progression of the disease, but permanent improvement is highly unlikely.

A client with Guillain-Barré syndrome develops respiratory acidosis as a result of reduced alveolar ventilation. Which combination of arterial blood gas (ABG) values confirms respiratory acidosis? a. pH, 7.25; PaCO2 50 mm Hg b. pH, 7.35; PaCO2 40 mm Hg c. pH, 7.40; PaCO2 35 mm Hg d. pH, 7.5; PaCO2 30 mm H

A In respiratory acidosis, ABG analysis reveals an arterial pH below 7.35 and partial pressure of arterial carbon dioxide (PaCO2) above 45 mm Hg. Therefore, the combination of a pH value of 7.25 and a PaCO2 value of 50 mm Hg confirms respiratory acidosis. A pH value of 7.5 with a PaCO2 value of 30 mm Hg indicates respiratory alkalosis. A ph value of 7.40 with a PaCO2 value of 35 mm Hg and a pH value of 7.35 with a PaCO2 value of 40 mm Hg represent normal ABG values, reflecting normal gas exchange in the lungs.

Which conditions lead to chronic respiratory acidosis in older adults? a. Thoracic skeletal change b. Overuse of sodium bicarbonate c. Decreased renal function d. Erratic meal patterns

A Poor respiratory exchange as the result of chronic lung disease, inactivity, or thoracic skeletal changes may lead to chronic respiratory acidosis. Decreased renal function in older adults can cause an inability to concentrate urine and is usually associated with fluid and electrolyte imbalance. A poor appetite, erratic meal patterns, inability to prepare nutritious meals, or financial circumstances may influence nutritional status, resulting in imbalances of electrolytes. Overuse of sodium bicarbonate may lead to metabolic alkalosis.

The position of a conscious client during suctioning is: A. Fowler's B. Supine position C. Side-lying D. Prone

A Position a conscious person who has a functional gag reflex in the semi fowler's position with the head turned to one side for oral suctioning or with the neck hyper extended for nasal suctioning. If the client is unconscious place the patient a lateral position facing you.

Which information obtained by the nurse when assessing a patient with acute respiratory distress syndrome (ARDS) who is being treated with mechanical ventilation and high levels of positive end-expiratory pressure (PEEP) indicates a complication of ventilator therapy is occurring? a. The patient has subcutaneous emphysema. b. The patient has a sinus bradycardia, rate 52. c. The patient's PaO2 is 50 mm Hg and the SaO2 is 88%. d. The patient has bronchial breath sounds in both the lung fields.

A Rationale: Complications of positive-pressure ventilation (PPV) and PEEP include subcutaneous emphysema. Bradycardia, hypoxemia, and bronchial breath sounds are all concerns, but they are not caused by PPV and PEEP.

A patient with acute respiratory distress syndrome (ARDS) has progressed to the fibrotic phase. The patient's family members are anxious about the patient's condition and are continuously present at the hospital. In addressing the family's concerns, it is important for the nurse to a. support the family and help them understand the realistic expectation that the patient's chance for survival is poor. b. inform the family that home health nurses will be able to help them maintain the mechanical ventilation at home after patient discharge. c. refer the family to social support services and case management to plan for transfer of the patient to a long-term care facility. d. provide hope and encouragement to the family because the patient's disease process has started to resolve.

A Rationale: The chance for survival is poor when the patient progresses to the fibrotic stage because permanent damage to the alveoli has occurred. Because of continued severe hypoxemia, the patient is not a candidate for home health or long-term care. The fibrotic stage indicates a poor patient prognosis, not the resolution of the ARDS process.

A client experiencing a severe asthma attack has the following arterial blood gas: pH 7.33; Pco2 48; Po2 58; HCO3 26. Which of the following orders should the nurse perform first? A. Albuterol (Proventil) nebulizer. B. Chest x-ray. C. Ipratropium (Atrovent) inhaler. D. Sputum culture

A The arterial blood gas reveals a respiratory acidosis with hypoxia. A quick-acting bronchodilator, albuterol, should be administered via nebulizer to improve gas exchange. Ipratropium is a maintenance treatment for bronchospasm that can be used with albuterol. A chest x-ray and sputum sample can be obtained once the client is stable.

28. While teaching a patient with asthma about the appropriate use of a peak flow meter, the nurse instructs the patient to do which of the following? A. Use the flow meter each morning after taking medications to evaluate their effectiveness. B. Empty the lungs and then inhale quickly through the mouthpiece to measure how fast air can be inhaled. C. Keep a record of the peak flow meter numbers if symptoms of asthma are getting worse. D. Increase the doses of the long-term control medication if the peak flow numbers decrease.

C. Keep a record of the peak flow meter numbers if symptoms of asthma are getting worse. It is important to keep track of peak flow readings daily and when the patient's symptoms are getting worse. The patient should have specific directions as to when to call the physician based on personal peak flow numbers. Peak flow is measured by exhaling into the meters and should be assessed before and after medications to evaluate their effectiveness.

20. During assessment of a 45-year-old patient with asthma, the nurse notes wheezing and dyspnea. The nurse interprets that these symptoms are related to which of the following pathophysiologic changes? A. Laryngospasm B. Overdistention of the alveoli C. Narrowing of the airway D. Pulmonary edema

C. Narrowing of the airwayNarrowing of the airway leads to reduced airflow, making it difficult for the patient to breathe and producing the characteristic wheezing.

11. When assessing a patient's respiratory status, which of the following nonrespiratory data are most important for the nurse to obtain? A. Height and weight B. Neck circumference C. Occupation and hobbies D. Usual daily fluid intake

C. Occupation and hobbiesMany respiratory problems occur as a result of chronic exposure to inhalation irritants. Common occupational sources of inhalation irritants include mines, granaries, farms, lawn care companies, paint, plastics and rubber manufacture, and building remodeling. Hobbies associated with inhalation irritants include woodworking, metal finishing, furniture refinishing, painting, and ceramics. Daily fluids, height, and weight are more related to respiratory problems secondary to cardiac issues.

26. Which of the following conditions is manifested by unexplained shortness of breath and a high mortality rate? A. Bleeding ulcer B. Transient ischemia C. Pulmonary embolism D. MI

C. Pulmonary embolism A high mortality rate is associated with a pulmonary embolism. A pulmonary embolism is an obstruction of the pulmonary artery caused by an embolus. It presents with hypoxia, anxiety, restlessness, and shortness of breath. Bleeding ulcers, MI, and transient ischemia are not associated with such a high mortality rate.

54. To promote airway clearance in a patient with pneumonia, the nurse instructs the patient to do which of the following? (Select all that apply.) A. Splint the chest when coughing B. Maintain a semi-Fowler's position C. Maintain adequate fluid intake D. Instruct patient to cough at end of exhalation

A,C,D The nurse should instruct the patient to splint the chest while coughing. This will reduce discomfort and allow for a more effective cough. Maintaining adequate fluid intake liquefies secretions, allowing easier expectoration. Coughing at the end of exhalation promotes a more effective cough. The patient should be positioned in an upright sitting position (high-Fowler's) with head slightly flexed.

Which of the following would be priority assessment data to gather from a client who has been diagnosed with pneumonia? Select all that apply. A. Auscultation of breath sounds B. Auscultation of bowel sounds C. .Presence of chest pain D. Presence of peripheral edema E. Color of nail beds

A,C,E A respiratory assessment, which includes auscultating breath sounds and assessing the color of the nail beds, is a priority for clients with pneumonia. Assessing for the presence of chest pain is also an important respiratory assessment as chest pain can interfere with the client's ability to breathe deeply. Auscultating bowel sounds and assessing for peripheral edema may be appropriate assessments, but these are not priority assessments for the patient with pneumonia.

The nurse is assigned to care for a patient after a left pneumonectomy. Which position is contraindicated for this patient? a. lateral b. low fowlers c. high fowlers d. elevated head of bed at 40 degrees.

A. Because there is no lung to stop mediastinal shift and compression of the remaining lung, lateral positioning is contraindicated.

29. The physician has prescribed salmeterol (Serevent) for a patient with asthma. In reviewing the use of dry powder inhalers (DPIs) with the patient, the nurse should provide which of the following instructions? A. "Close lips tightly around the mouthpiece and breathe in deeply and quickly." B. "To administer a DPI, you must use a spacer that holds the medicine so that you can inhale it." C. "Hold the inhaler several inches in front of your mouth and breathe in slowly, holding the medicine as long as possible." D. "You will know you have correctly used the DPI when you taste or sense the medicine going into your lungs."

A. "Close lips tightly around the mouthpiece and breathe in deeply and quickly." Dry powder inhalers do not require spacer devices. The patient should be instructed to breathe in deeply and quickly to ensure medicine moves down deeply into lungs. The patient may not taste or sense the medicine going into the lungs.

32. Which of the following test results identify that a patient with an asthma attack is responding to treatment? A. A decreased exhaled nitric oxide B. An increase in CO2 levels C. A decrease in white blood cell count D. An increase in serum bicarbonate levels

A. A decreased exhaled nitric oxide. Nitric oxide levels are increased in the breath of people with asthma. A decrease in the exhaled nitric oxide concentration suggests that the treatment may be decreasing the lung inflammation associated with asthma.

4. When caring for a patient who is 3 hours postoperative laryngectomy, the nurse's highest priority assessment would be: A. Airway patency B. Patient comfort C. Incisional drainage D. Blood pressure and heart rate

A. Airway patency Remember ABCs with prioritization. Airway patency is always the highest priority and is essential for a patient undergoing surgery surrounding the upper respiratory system.

34. The patient has an order for each of the following inhalers. Which of the following should the nurse offer to the patient at the onset of an asthma attack? A. Albuterol (Proventil) B. Beclomethasone (Beclovent) C. Ipratropium bromide (Atrovent) D. Salmeterol (Serevent)

A. Albuterol (Proventil) Albuterol is a short-acting bronchodilator that should initially be given when the patient experiences an asthma attack.

21. A 45-year-old man with asthma is brought to the emergency department by automobile. He is short of breath and appears frightened. During the initial nursing assessment, which of the following clinical manifestations might be present as an early symptom during an exacerbation of asthma? A. Anxiety B. Cyanosis C. Hypercapnia D. Bradycardia

A. Anxiety An early symptom during an asthma attack is anxiety because he is acutely aware of the inability to get sufficient air to breathe. He will be hypoxic early on with decreased PaCO2 and increased pH as he is hyperventilating.

12. If a nurse is assessing a patient whose recent blood gas determination indicated a pH of 7.32 and respirations are measured at 32 breaths/min, which of the following is the most appropriate nursing assessment? A. The rapid breathing is causing the low pH. B. The nurse should sedate the patient to slow down respirations. C. The rapid breathing is an attempt to compensate for the low pH. D. The nurse should give the patient a paper bag to breathe into to correct the low pH.

C. The rapid breathing is an attempt to compensate for the low pH. The respiratory system influences pH (acidity) through control of carbon dioxide exhalation. Thus, rapid breathing increases the pH. Breathing into a paper bag aids a patient who is hyperventilating; in respiratory alkalosis, it aids in lowering the pH. The use of sedation can cause respiratory depression and hypoventilation, resulting in an even lower pH.

The nurse is gathering data on a patient with a suspected diagnosis of tuberculosis. Which of the following tests will confirm this diagnosis? a. chest x ray b. bronchoscopy c. sputum culture d. tuberculin skin test

C. a sputum culture positive for mycobacterium tuberculosis the definitive diagnosis. A presumptive diagnosis is made on any of the other three.

6. The nurse is caring for a patient admitted to the hospital with pneumonia. Upon assessment, the nurse notes a temperature of 101.4° F, a productive cough with yellow sputum and a respiratory rate of 20. Which of the following nursing diagnosis is most appropriate based upon this assessment? A. Hyperthermia related to infectious illness B. Ineffective thermoregulation related to chilling C. Ineffective breathing pattern related to pneumonia D. Ineffective airway clearance related to thick secretions

A. Hyperthermia related to infectious illness Because the patient has spiked a temperature and has a diagnosis of pneumonia, the logical nursing diagnosis is hyperthermia related to infectious illness. There is no evidence of a chill, and her breathing pattern is within normal limits at 20 breaths per minute. There is no evidence of ineffective airway clearance from the information given because the patient is expectorating sputum.

35. The nurse who has administered a first dose of oral prednisone (Deltasone) to the patient with asthma writes on the care plan to begin monitoring which of the following patient parameters? A. Intake and output B. Bowel sounds C. Apical pulse D. Deep tendon reflexes

A. Intake and output Corticosteroids such as prednisone can lead to fluid retention. For this reason, it is important to monitor the patient's intake and output.

10. If a nurse is caring for an 80-year-old patient with a temperature of 100.4° F, crackles at the right lung base, pain with deep inspiration, and dyspnea, which of the following orders is the nurse's priority? A. Sputum specimen for culture and sensitivity B. Codeine 15 mg orally every 6 hours as needed C. Incentive spirometer every 2 hours while awake D. Amoxicillin (Amoxil) 500 mg orally 4 times a day

A. Sputum specimen for culture and sensitivity The patient presents with signs of a respiratory infection. To initiate the most effective therapy, the health care prescriber must know the pathogen causing the infection. Therefore, the sputum specimen is the nurse's priority. If the antibiotic is administered before the specimen is obtained, the results of the culture might not be as accurate and could impair the effectiveness of therapy. After the specimen is obtained, the nurse can administer codeine for coughing and begin the incentive spirometry to mobilize secretions and improve the patient's ability to expectorate the secretions.

27. A patient with acute exacerbation of COPD needs to receive precise amounts of oxygen. Which of the following types of equipment should the nurse prepare to use? A. Venturi mask B. Partial non-rebreather mask C. Oxygen tent D. Nasal cannula

A. Venturi mask The Venturi mask delivers precise concentrations of oxygen and should be selected whenever this is a priority concern. The other methods are less precise in terms of amount of oxygen delivered.

33. A patient with an acute pharyngitis is seen at the clinic with fever and severe throat pain that affects swallowing. On inspection the throat is reddened and edematous with patchy yellow exudates. The nurse anticipates that collaborative management will include A. treatment with antibiotics. B. treatment with antifungal agents. C. a throat culture or rapid strep antigen test. D. treatment with medication only if the pharyngitis does not resolve in 3 to 4 days.

C. a throat culture or rapid strep antigen test. Although inadequately treated β-hemolytic streptococcal infections may lead to rheumatic heart disease or glomerulonephritis, antibiotic treatment is not recommended until strep infections are definitely diagnosed with culture or antigen tests. The manifestations of viral and bacterial infections are similar, and appearance is not diagnostic except when candidiasis is present.

32. An excess of carbon dioxide in the blood causes an increased respiratory rate and volume because CO2 A. displaces oxygen on hemoglobin, leading to a decreased PaO2. B. causes an increase in the amount of hydrogen ions available in the body. C. combines with water to form carbonic acid, lowering the pH of cerebrospinal fluid. D. directly stimulates chemoreceptors in the medulla to increase respiratory rate and volume.

C. combines with water to form carbonic acid, lowering the pH of cerebrospinal fluid. A combination of excess CO2 and H2O results in carbonic acid, which lowers the pH of the cerebrospinal fluid and stimulates an increase in the respiratory rate. Peripheral chemoreceptors in the carotid and aortic bodies also respond to increases in PaCO2 to stimulate the respiratory center. Excess CO2 does not increase the amount of hydrogen ions available in the body but does combine with the hydrogen of water to form an acid.

34. Following a supraglottic laryngectomy, the patient is taught how to use the supraglottic swallow to minimize the risk of aspiration. In teaching the patient about this technique, the nurse instructs the patient to A. perform Valsalva maneuver immediately after swallowing. B. breathe between each Valsalva maneuver and cough sequence. C. cough after swallowing to remove food from the top of the vocal cords. D. practice swallowing thin, watery fluids before attempting to swallow solid foods.

C. cough after swallowing to remove food from the top of the vocal cords. A supraglottic laryngectomy involves removal of the epiglottis and false vocal cords, and the removal of the epiglottis allows food to enter the trachea. Supraglottic swallowing requires performance of the Valsalva maneuver before placing food in the mouth and swallowing. The patient then coughs to remove food from the top of the vocal cords, swallows again, and then breathes after the food has been removed from the vocal cords.

18. A nurse establishes the presence of a tension pneumothorax when assessment findings reveal a(n) A. absence of lung sounds on the affected side. B. inability to auscultate tracheal breath sounds. C. deviation of the trachea toward the side opposite the pneumothorax. D. shift of the point of maximal impulse (PMI) to the left, with bounding pulses.

C. deviation of the trachea toward the side opposite the pneumothorax. Tension pneumothorax is caused by rapid accumulation of air in the pleural space, causing severely high intrapleural pressure. This results in collapse of the lung, and the mediastinum shifts toward the unaffected side, which is subsequently compressed.

37. The chronic inflammation of the bronchi characteristic of chronic obstructive pulmonary disease (COPD) results in A. collapse of small bronchioles on expiration. B. permanent, abnormal dilation of the bronchi. C. hyperplasia of mucus-secreting cells and bronchial edema. D. destruction of the elastic and muscular structures of the bronchial wall.

C. hyperplasia of mucus-secreting cells and bronchial edema. Chronic bronchitis is characterized by chronic inflammation of the bronchial lining, with edema and increased mucus production. Collapse of small bronchioles on expiration is common in emphysema, and abnormal dilation of the bronchi because of destruction of the elastic and muscular structures is characteristic of bronchiectasis.

9. In older adults, infection after exposure to respiratory illness is most likely to A. result in similar rates of infection as in the younger adult. B. be easily prevented with the use of antibiotics after being exposed. C. result in serious lower respiratory infection related to weakened respiratory muscles and fewer cilia. D. be less serious because the older adult has less contact with younger children who are most likely to carry serious infections.

C. result in serious lower respiratory infection related to weakened respiratory muscles and fewer cilia. Changes in the older adult respiratory system make older adults more susceptible to infections that can be very serious and life threatening. Use of antibiotics to "prevent" lung infections is not recommended and is ineffective for viral infections.

53. A patient is being discharged from the emergency department after being treated for epistaxis. In teaching the family first aid measures in the event the epistaxis would recur, which of the following measures would the nurse suggest? (Select all that apply.) A. Tilt patients head backwards B. Apply ice compresses to the nose C. Pinch the entire soft lower portion of the nose D. Partially insert a small gauze pad into the bleeding nostril

B,C,D First aid measures to control epistaxis includes placing the patient in a sitting position, leaning forward. Tilting the head back does not stop the bleeding, but rather allows the blood to enter the nasopharynx, which could result in aspiration or nausea/vomiting from swallowing blood. All of the other options are appropriate first aid treatment of epistaxis.

The nurse is teaching a client who has been diagnosed with TB how to avoid spreading the disease to family members. Which statement(s) by the client indicate(s) that he has understood the nurses instructions? Select all that apply. A. "I will need to dispose of my old clothing when I return home." B. "I should always cover my mouth and nose when sneezing." C. "It is important that I isolate myself from family when possible." D. "I should use paper tissues to cough in and dispose of them properly." E. "I can use regular plate and utensils whenever I eat."

B,D,E

The nurse is reading the results of a tuberculin skin test on a patient with no documented health problems. The site has no induration and a 1mm area of ecchymosis. Which interpretation should the nurse make of these results? a. positive b. negative c. uncertain d. borderline

B. a positive test has an induration measuring 10mm or more. the ecchymosis is insignificant.

50. The nurse is teaching a patient how to self-administer ipratropium (Atrovent) via a metered dose inhaler. Which of the following instructions given by the nurse is most appropriate to help the patient learn proper inhalation technique? A. "Avoid shaking the inhaler before use." B. "Breathe out slowly before positioning the inhaler." C. "After taking a puff, hold the breath for 30 seconds before exhaling." D. "Using a spacer should be avoided for this type of medication."

B. "Breathe out slowly before positioning the inhaler." It is important to breathe out slowly before positioning the inhaler. This allows the patient to take a deeper breath while inhaling the medication thus enhancing the effectiveness of the dose.

47. A patient has been receiving oxygen per nasal cannula while hospitalized for COPD. The patient asks the nurse whether oxygen use will be needed at home. Which of the following would be the most appropriate response by the nurse? A. "Long-term home oxygen therapy should be used to prevent respiratory failure." B. "Oxygen will be needed when your oxygen saturation drops to 88% and you have symptoms of hypoxia. C. "Long-term home oxygen therapy should be used to prevent heart problems related to emphysema." D. "Oxygen will not be needed until or unless you are in the terminal stages of this disease."

B. "Oxygen will be needed when your oxygen saturation drops to 88% and you have symptoms of hypoxia.Long-term oxygen therapy in the home should be considered when the oxygen saturation is 88% or less and the patient has signs of tissue hypoxia, such as cor pulmonale, erythrocytosis, or impaired mental status.

20. If a patient states, "It's hard for me to breathe and I feel short-winded all the time," what is the most appropriate terminology to be applied in documenting this assessment by a nurse? A. Apnea B. Dyspnea C. Tachypnea D. Respiratory fatigue

B. Dyspnea Dyspnea is a subjective description reflective of the patient's statement indicating difficulty in breathing. Apnea refers to absence of breath or breathing. Tachypnea refers to an increased rate of breathing, usually greater than 20 breaths per minute. Respiratory fatigue is subjective and usually refers to the patient exhibiting signs and symptoms associated with a comprehensive respiratory assessment including laborious breathing, use of accessory muscles, and slowing of respirations.

9. Which of the following nursing interventions is of the highest priority in helping a patient expectorate thick secretions related to pneumonia? A. Humidify the oxygen as able B. Increase fluid intake to 3L/day if tolerated. C. Administer cough suppressant q4hr. D. Teach patient to splint the affected area.

B. Increase fluid intake to 3L/day if tolerated. Although several interventions may help the patient expectorate mucus, the highest priority should be on increasing fluid intake, which will liquefy the secretions so that the patient can expectorate them more easily. Humidifying the oxygen is also helpful, but is not the primary intervention. Teaching the patient to splint the affected area may also be helpful, but does not liquefy the secretions so that they can be removed.

26. The nurse is assigned to care for a patient in the emergency department admitted with an exacerbation of asthma. The patient has received a β-adrenergic bronchodilator and supplemental oxygen. If the patient's condition does not improve, the nurse should anticipate which of the following is likely to be the next step in treatment? A. Pulmonary function testing B. Systemic corticosteroids C. Biofeedback therapy D. Intravenous fluids

B. Systemic corticosteroids Systemic corticosteroids speed the resolution of asthma exacerbations and are indicated if the initial response to the β-adrenergic bronchodilator is insufficient.

48. Before discharge, the nurse discusses activity levels with a 61-year-old patient with COPD and pneumonia. Which of the following exercise goals is most appropriate once the patient is fully recovered from this episode of illness? A. Slightly increase activity over the current level. B. Walk for 20 minutes a day, keeping the pulse rate less than 130 beats per minute. C. Limit exercise to activities of daily living to conserve energy. D. Swim for 10 min/day, gradually increasing to 30 min/day.

B. Walk for 20 minutes a day, keeping the pulse rate less than 130 beats per minute. The patient will benefit from mild aerobic exercise that does not stress the cardiorespiratory system. The patient should be encouraged to walk for 20 min/day, keeping the pulse rate less than 75% to 80% of maximum heart rate (220 minus patient's age).

6. A nurse is working on a respiratory care unit where many of the patients are affected by asthma. Which of the following actions by the nurse would most likely increase respiratory difficulty for the patients? A. Wearing perfume to work B. Encouraging patients to ambulate daily C. Allowing the patients to eat green leafy vegetables D. Withholding antibiotic therapy until cultures are obtained

A. Wearing perfume to work People with asthma should avoid extrinsic allergens and irritants (e.g., dust, pollen, smoke, certain foods, colognes and perfumes, certain types of medications) because their airways become inflamed, producing shortness of breath, chest tightness, and wheezing. Many green leafy vegetables are rich in vitamins, minerals, and proteins, which incorporate healthy lifestyle patterns into the patients' daily living routines. Routine exercise is a part of a prudent lifestyle, and for patients with asthma the physical and psychosocial effects of ambulation can incorporate feelings of well-being, strength, and enhancement of physical endurance. Antibiotic therapy is always initiated after cultures are obtained so that the sensitivity to the organism can be readily identified.

24. The nurse is caring for a patient with an acute exacerbation of asthma. Following initial treatment, which of the following findings indicates to the nurse that the patient's respiratory status is improving? A. Wheezing becomes louder B. Vesicular breath sounds decrease C. Aerosol bronchodilators stimulate coughing D. The cough remains nonproductive

A. Wheezing becomes louder The primary problem during an exacerbation of asthma is narrowing of the airway and subsequent diminished air exchange. As the airways begin to dilate, wheezing gets louder because of better air exchange.

36. The resurgence in TB resulting from the emergence of multidrug-resistant strains of Mycobacterium tuberculosis is primarily the result of A. a lack of effective means to diagnose TB. B. poor compliance with drug therapy in patients with TB. C. the increased population of immunosuppressed individuals with AIDS. D. indiscriminate use of antitubercular drugs in treatment of other infections.

B. poor compliance with drug therapy in patients with TB. Drug-resistant strains of TB have developed because TB patients' compliance to drug therapy has been poor and there has been general decreased vigilance in monitoring and follow-up of TB treatment. Antitubercular drugs are almost exclusively used for TB infections. TB can be effectively diagnosed with sputum cultures. The incidence of TB is at epidemic proportions in patients with HIV, but this does not account for drug-resistant strains of TB.

2. A patient admitted to the emergency department with tension pneumothorax and mediastinal shift following an automobile crash is most likely to exhibit A. bradycardia. B. severe hypotension. C. mediastinal flutter. D. a sucking chest wound.

B. severe hypotension. Mediastinal shift may cause compression of the lung in the direction of the shift and compression, traction, torsion, or kinking of the great vessels. Blood return to the heart is dangerously impaired and causes a subsequent decrease in cardiac output and blood pressure. Tachycardia is a clinical manifestation of tension pneumothorax. An uncovered opened pneumothorax is associated with a sucking chest wound and mediastinal flutter.

25. A 75-year-old obese patient who is snoring loudly and having periods of apnea several times each night is most likely experiencing A. narcolepsy. B. sleep apnea. C. sleep deprivation. D. paroxysmal nocturnal dyspnea.

B. sleep apnea. Sleep apnea is most common in obese patients. Typical symptoms include snoring and periods of apnea. Narcolepsy is when a patient falls asleep unexpectedly. Sleep deprivation could result from sleep apnea. Paroxysmal nocturnal dyspnea occurs when a patient has shortness of breath during the night.

A pediatric patient has been diagnosed with right lower lobe pneumonia. Upon auscultation of this lung field, the healthcare provider would expect to hear which breath sound? Please choose from one of the following options. A. Rhonchi B.Stridor C. Crackles D. Wheezes

C

The healthcare provider is caring for a patient on a ventilator with an endotracheal tube in place. What assessment data indicate the tube has migrated too far down the trachea? Please choose from one of the following options. A. Increased crackles auscultation bilaterally B. A high pressure alarm sounds C. Decreased breath sounds on the left side of the chest D. Low pressure alarm sounds

C

pH 7.4, PaCO2 59, HCO3- 35 A. Respiratory Acidosis, Uncompensated B. Metabolic Alkalosis, Uncompensated C. Respiratory Acidosis, Fully Compensated D. Metabolic Alkalosis, Partially Compensated

C

Which of the following methods is the best way to confirm the diagnosis of a pneumothorax? A. Auscultate breath sounds B. Have the client use an incentive spirometer C. Take a chest x-ray D. Stick a needle in the area of decreased breath sounds

C A chest x-ray will show the area of collapsed lung if pneumothorax is present as well as the volume of air in the pleural space. Listening to breath sounds won't confirm a diagnosis. An IS is used to encourage deep breathing. A needle thoracostomy is done only in an emergency and only by someone trained to do it.

Baby Angela was rushed to the Emergency Room following her mother's complaint that the infant has been irritable, difficult to breastfeed and has had diarrhea for the past 3 days. The infant's respiratory rate is elevated and the fontanels are sunken. The Emergency Room physician orders ABGs after assessing the ABCs. The results from the ABG results show pH 7.39, PaCO2 27 mmHg and HCO3 19 mEq/L. What does this mean? A. Respiratory Alkalosis, Fully Compensated B. .Metabolic Acidosis, Uncompensated C. Metabolic Acidosis, Fully Compensated D. Respiratory Acidosis, Uncompensated

C Baby Angela has metabolic acidosis due to decreased HCO3 and slightly acidic pH. Her pH value is within the normal range which made the result fully compensated.

A client with pneumonia has a temperature of 102.6*F (39.2*C), is diaphoretic, and has a productive cough. The nurse should include which of the following measures in the plan of care? A. Position changes q4h B. Nasotracheal suctioning to clear secretions C. Frequent linen changes D. Frequent offering of a bedpan

C Frequent linen changes are appropriate for this client because of diaphoresis. Diaphoresis produces general discomfort. The client should be kept dry to promote comfort. Position changes need to be done every 2 hours. Nasotracheal suctioning is not indicated with the client's productive cough. Frequent offering of a bedpan is not indicated by the data provided in this scenario.

Increased plasma bicarbonate (HCO3-) A. Metabolic Acidosis B. Respiratory Alkalosis C. Metabolic Alkalosis D. Respiratory Acidosis

C In metabolic alkalosis, breathing becomes depressed in an effort to conserve carbon dioxide for combination with water in the blood to raise the blood level of carbonic acid. Symptoms include confusion, dizziness, numbness or tingling of fingers or toes.

16. To ensure the correct amount of oxygen delivery for a patient receiving 35% oxygen via a Venturi mask, it is most important that the nurse A. keep the air-entrainment ports clean and unobstructed. B. apply an adaptor to increase humidification of the oxygen. C. drain moisture condensation from the oxygen tubing every hour. D. keep the flow rate high enough to keep the bag from collapsing during inspiration.

A. keep the air-entrainment ports clean and unobstructed. Oxygen is delivered to a small jet in the center of a wide-based cone. Air is entrained (pulled through) openings in the cone as oxygen flows through the small jet. The degree of restriction or narrowness of the jet determines the amount of entrainment and the dilution of pure oxygen with room air and thus the concentration of oxygen. Although applying an adaptor can increase the humidification with the Venturi mask, it is not the best answer, because an open port is essential to proper functioning. Draining moisture condensation from the oxygen tubing is performed as often as needed, not on an hourly schedule. A plastic face mask with a reservoir bag needs to have sufficient flow rate to keep the bag inflated.

17. While caring for a patient with respiratory disease, a nurse observes that the oxygen saturation drops from 94% to 85% when the patient ambulates. The nurse should determine that A. supplemental oxygen should be used when the patient exercises. B. ABG determinations should be done to verify the oxygen saturation reading. C. this finding is a normal response to activity and that the patient should continue to be monitored. D. the oximetry probe should be moved from the finger to the earlobe for an accurate oxygen saturation measurement during activity.

A. supplemental oxygen should be used when the patient exercises.An oxygen saturation lower than 90% indicates inadequate oxygenation. If the drop is related to activity of some type, supplemental oxygen is indicated.

An 87-year-old client requires long term ventilator therapy. He has a tracheostomy in place and requires frequent suctioning. Which of the following techniques is correct? A. Using intermittent suction while advancing the catheter B. Using continuous suction while withdrawing the catheter C. Using intermittent suction while withdrawing the catheter D. Using continuous suction while advancing the catheter

C Intermittent suction should be applied during catheter withdrawal. To prevent hypoxia, suctioning shouldn't last more than 10-seconds at a time. Suction shouldn't be applied while the catheter is being advanced.

An old beggar was admitted to the emergency department due to shortness of breath, fever, and a productive cough. Upon examination, crackles and wheezes are noted in the lower lobes; he appears to be tachycardic and has a bounding pulse. Measurement of arterial blood gas shows pH 7.2, PaCO2 66 mm Hg, HCO3 27 mmol/L, and PaO2 65 mm Hg. As a knowledgeable nurse, you know that the normal value for pH is: A. 7.20 B. 7.30 C. 7.40 D. 7.50

C Normal blood pH must be maintained within a narrow range of 7.35-7.45 to ensure the proper functioning of metabolic processes and the delivery of the right amount of oxygen to tissues. Acidosis refers to an excess of acid in the blood that causes the pH to fall below 7.35, and alkalosis refers to an excess of base in the blood that causes the pH to rise above 7.45.

Anne, who is drinking beer at a party, falls and hits her head on the ground. Her friend Liza dials "911" because Anne is unconscious, depressed ventilation (shallow and slow respirations), rapid heart rate, and is profusely bleeding from both ears. Which primary acid-base imbalance is Anne at risk for if medical attention is not provided? A. Metabolic Acidosis B. Metabolic Alkalosis C. Respiratory Acidosis D. Respiratory Alkalosis

C One of the risk factors of having respiratory acidosis is hypoventilation which may be due to brain trauma, coma, and hypothyroidism or myxedema. Other risk factors include COPD, Respiratory conditions such as pneumothorax, pneumonia and status asthmaticus. Drugs such as Morphine and MgSO4 toxicity are also risk factors of respiratory acidosis.

A patient is admitted to the emergency department complaining of sudden onset shortness of breath and is diagnosed with a possible pulmonary embolus. How should the nurse prepare the patient for diagnostic testing to confirm the diagnosis? a. Start an IV so contrast media may be given. b. Ensure that the patient has been NPO for at least 6 hours. c. Inform radiology that radioactive glucose preparation is needed. d. Instruct the patient to undress to the waist and remove any metal objects.

ANS: A Spiral computed tomography (CT) scans are the most commonly used test to diagnose pulmonary emboli, and contrast media may be given IV. A chest x-ray may be ordered but will not be diagnostic for a pulmonary embolus. Preparation for a chest x-ray includes undressing and removing any metal. Bronchoscopy is used to detect changes in the bronchial tree, not to assess for vascular changes, and the patient should be NPO 6 to 12 hours before the procedure. Positron emission tomography (PET) scans are most useful in determining the presence of malignancy, and a radioactive glucose preparation is used.

The nurse assesses a patient with chronic obstructive pulmonary disease (COPD) who has been admitted with increasing dyspnea over the last 3 days. Which finding is most important for the nurse to report to the health care provider? a. Respirations are 36 breaths/minute. b. Anterior-posterior chest ratio is 1:1. c. Lung expansion is decreased bilaterally. d. Hyperresonance to percussion is present.

ANS: A The increase in respiratory rate indicates respiratory distress and a need for rapid interventions such as administration of oxygen or medications. The other findings are common chronic changes occurring in patients with COPD.

A diabetic patient's arterial blood gas (ABG) results are pH 7.28; PaCO2 34 mm Hg; PaO2 85 mm Hg; HCO3- 18 mEq/L. The nurse would expect which finding? a. Intercostal retractions b. Kussmaul respirations c. Low oxygen saturation (SpO2) d. Decreased venous O2 pressure

ANS: B Kussmaul (deep and rapid) respirations are a compensatory mechanism for metabolic acidosis. The low pH and low bicarbonate result indicate metabolic acidosis. Intercostal retractions, a low oxygen saturation rate, and a decrease in venous O2 pressure would not be caused by acidosis.

A patient in metabolic alkalosis is admitted to the emergency department, and pulse oximetry (SpO2) indicates that the O2 saturation is 94%. Which action should the nurse take next? a. Administer bicarbonate. b. Complete a head-to-toe assessment. c. Place the patient on high-flow oxygen. d. Obtain repeat arterial blood gases (ABGs).

ANS: C Although the O2 saturation is adequate, the left shift in the oxyhemoglobin dissociation curve will decrease the amount of oxygen delivered to tissues, so high oxygen concentrations should be given. Bicarbonate would worsen the patient's condition. A head-to-toe assessment and repeat ABGs may be implemented. However, the priority intervention is to give high-flow oxygen.

The nurse teaches a patient about pulmonary function testing (PFT). Which statement, if made by the patient, indicates teaching was effective? a. "I will use my inhaler right before the test." b. "I won't eat or drink anything 8 hours before the test." c. "I should inhale deeply and blow out as hard as I can during the test." d. "My blood pressure and pulse will be checked every 15 minutes after the test."

ANS: C For PFT, the patient should inhale deeply and exhale as long, hard, and fast as possible. The other actions are not needed with PFT. The administration of inhaled bronchodilators should be avoided 6 hours before the procedure.

The nurse analyzes the results of a patient's arterial blood gases (ABGs). Which finding would require immediate action? a. The bicarbonate level (HCO3-) is 31 mEq/L. b. The arterial oxygen saturation (SaO2) is 92%. c. The partial pressure of CO2 in arterial blood (PaCO2) is 31 mm Hg. d. The partial pressure of oxygen in arterial blood (PaO2) is 59 mm Hg.

ANS: D All the values are abnormal, but the low PaO2 indicates that the patient is at the point on the oxyhemoglobin dissociation curve where a small change in the PaO2 will cause a large drop in the O2 saturation and a decrease in tissue oxygenation. The nurse should intervene immediately to improve the patient's oxygenation.

Liza's mother is seen in the emergency department at a community hospital. She admits that her mother is taking many tablets of aspirin (salicylates) over the last 24-hour period because of a severe headache. Also, the mother complains of an inability to urinate. The nurse on duty took her vital signs and noted the following: Temp = 97.8 °F; apical pulse = 95; respiration = 32 and deep. Which primary acid-base imbalance is the gentleman at risk for if medical attention is not provided? A. Respiratory Acidosis B. Respiratory Alkalosis C. Metabolic Acidosis D. Metabolic Alkalosis

C Salicylate overdose causes a high anion gap metabolic acidosis in both children and adults. Adults commonly develop a mixed acid-base disorder as a respiratory alkalosis due to direct respiratory centre stimulation occurs as well. This second disorder is uncommon in children.

A client with pneumonia develops dyspnea with a respiratory rate of 32 breaths/minute and difficulty expelling his secretions. The nurse auscultates his lung fields and hears bronchial sounds in the left lower lobe. The nurse determines that the client requires which of the following treatments first? A. Antibiotics B. Bed rest C. Oxygen D. Nutritional intake

C The client is having difficulty breathing and is probably becoming hypoxic. As an emergency measure, the nurse can provide oxygen without waiting for a physicians order. Antibiotics may be warranted, but this isn't a nursing decision. The client should be maintained on bedrest if he is dyspneic to minimize his oxygen demands, but providing additional will deal more immediately with his problem. The client will need nutritional support, but while dyspneic, he may be unable to spare the energy needed to eat and at the same time maintain adequate oxygenation.

A client diagnosed with active TB would be hospitalized primarily for which of the following reasons? A. To evaluate his condition B. To determine his compliance C. To prevent spread of the disease D. To determine the need for antibiotic therapy

C The client with active TB is highly contagious until three consecutive sputum cultures are negative, so he's put in respiratory isolation in the hospital.

Continuous positive airway pressure (CPAP) can be provided through an oxygen mask to improve oxygenation in hypoxic patients by which of the following methods? A. The mask provides 100% oxygen to the client B. The mask provides continuous air that the client can breathe C. The mask provides pressurized oxygen so the client can breathe more easily D. The mask provides pressurized at the end of expiration to open collapsed alveoli

C The mask provides pressurized oxygen continuously through both inspiration and expiration. The mask can be set to deliver any amount of oxygen needed. By providing the client with pressurized oxygen, the client has less resistance to overcome in taking his next breath, making it easier to breathe. Pressurized oxygen delivered at the end of expiration is positive end-expiratory pressure (PEEP), not continuous positive airway pressure.

Ricky's grandmother is suffering from persistent vomiting for two days now. She appears to be lethargic and weak and has myalgia. She is noted to have dry mucus membranes and her capillary refill takes >4 seconds. She is diagnosed as having gastroenteritis and dehydration. Measurement of arterial blood gas shows pH 7.5, PaO2 85 mm Hg, PaCO2 40 mm Hg, and HCO3 34 mmol/L. What acid-base disorder is shown? A. Respiratory Alkalosis, Uncompensated B. Respiratory Acidosis, Partially Compensated C. Metabolic Alkalosis, Uncompensated D. Metabolic Alkalosis, Partially Compensated

C The primary disorder is uncompensated metabolic alkalosis (high HCO3 -). As CO2 is the strongest driver of respiration, it generally will not allow hypoventilation as compensation for metabolic alkalosis.

Client Z is admitted to the hospital and is to undergo brain surgery. The client is very anxious and scared of the upcoming surgery. He begins to hyperventilate and becomes very dizzy. The client loses consciousness and the STAT ABGs reveal pH 7.61, PaCO2 22 mmHg and HCO3 25 mEq/L. What is the ABG interpretation based on the findings? A. Metabolic Acidosis, Uncompensated B. Respiratory Alkalosis, Partially Compensated C. Respiratory Alkalosis, Uncompensated D. Metabolic Alkalosis, Partially Compensated

C The results show that client Z has respiratory alkalosis since there is an increase in the pH value and a decrease in PaCO2 which are both basic. It is uncompensated due to the normal HCO3 which is within 22-26 mEq/L.

Which of the following diagnostic tests is definitive for TB? A. Chest x-ray B. Mantoux test C. Sputum culture D. Tuberculin test

C The sputum culture for Mycobacterium tuberculosis is the only method of confirming the diagnosis. Lesions in the lung may not be big enough to be seen on x-ray. Skin tests may be falsely positive or falsely negative.

The right forearm of a client who had a purified protein derivative (PPD) test for tuberculosis is reddened and raised about 3mm where the test was given. This PPD would be read as having which of the following results? A. Indeterminate B. Needs to be redone C. Negative D. Positive

C This test would be classed as negative. A 5 mm raised area would be a positive result if a client was HIV+ or had recent close contact with someone diagnosed with TB. Indeterminate isn't a term used to describe results of a PPD test. If the PPD is reddened and raised 10mm or more, it's considered positive according to the CDC.

. A patient who has a history of chronic obstructive pulmonary disease (COPD) was hospitalized for increasing shortness of breath and chronic hypoxemia (SaO2 levels of 89% to 90%). In planning for discharge, which action by the nurse will be most effective in improving compliance with discharge teaching? a. Start giving the patient discharge teaching on the day of admission. b. Have the patient repeat the instructions immediately after teaching. c. Accomplish the patient teaching just before the scheduled discharge. d. Arrange for the patient's caregiver to be present during the teaching.

ANS: D Hypoxemia interferes with the patient's ability to learn and retain information, so having the patient's caregiver present will increase the likelihood that discharge instructions will be followed. Having the patient repeat the instructions will indicate that the information is understood at the time, but it does not guarantee retention of the information. Because the patient is likely to be distracted just before discharge, giving discharge instructions just before discharge is not ideal. The patient is likely to be anxious and even more hypoxemic than usual on the day of admission, so teaching about discharge should be postponed.

The nurse admits a patient who has a diagnosis of an acute asthma attack. Which statement indicates that the patient may need teaching regarding medication use? a. "I have not had any acute asthma attacks during the last year." b. "I became short of breath an hour before coming to the hospital." c. "I've been taking Tylenol 650 mg every 6 hours for chest-wall pain." d. "I've been using my albuterol inhaler more frequently over the last 4 days."

ANS: D The increased need for a rapid-acting bronchodilator should alert the patient that an acute attack may be imminent and that a change in therapy may be needed. The patient should be taught to contact a health care provider if this occurs. The other data do not indicate any need for additional teaching.

Using the illustrated technique, the nurse is assessing for which finding in a patient with chronic obstructive pulmonary disease (COPD)? a. Hyperresonance b. Tripod positioning c. d. Reduced chest expansion

ANS: D The technique for palpation for chest expansion is shown in the illustrated technique. Reduced chest movement would be noted on palpation of a patient's chest with COPD. Hyperresonance would be assessed through percussion. Accessory muscle use and tripod positioning would be assessed by inspection.

The nurse prepares a patient with a left-sided pleural effusion for a thoracentesis. How should the nurse position the patient? a. Supine with the head of the bed elevated 30 degrees b. In a high-Fowler's position with the left arm extended c. On the right side with the left arm extended above the head d. Sitting upright with the arms supported on an over bed table

ANS: D The upright position with the arms supported increases lung expansion, allows fluid to collect at the lung bases, and expands the intercostal space so that access to the pleural space is easier. The other positions would increase the work of breathing for the patient and make it more difficult for the health care provider performing the thoracentesis.

The laboratory has just called with the arterial blood gas (ABG) results on four patients. Which result is most important for the nurse to report immediately to the health care provider? a. pH 7.34, PaO2 82 mm Hg, PaCO2 40 mm Hg, and O2 sat 97% b. pH 7.35, PaO2 85 mm Hg, PaCO2 45 mm Hg, and O2 sat 95% c. pH 7.46, PaO2 90 mm Hg, PaCO2 32 mm Hg, and O2 sat 98% d. pH 7.31, PaO2 91 mm Hg, PaCO2 50 mm Hg, and O2 sat 96%

ANS: D These ABGs indicate uncompensated respiratory acidosis and should be reported to the health care provider. The other values are normal or close to normal.

18. Which of the following respiratory disorders is most common in the first 24 to 48 hours after surgery? a. Atelectasis b. Bronchitis c. Pneumonia d. Pneumothorax

ANSWER A. Atelectasis develops when there's interference with the normal negative pressure that promotes lung expansion. Clients in the postoperative phase often splint their breathing because of pain and positioning, which causes hypoxia. It's uncommon for any of the other respiratory disorders to develop.

13. Exercise has which of the following effects on clients with asthma, chronic bronchitis, and emphysema? a. It enhances cardiovascular fitness. b. It improves respiratory muscle strength. c. It reduces the number of acute attacks. d. It worsens respiratory function and is discouraged.

ANSWER A. Exercise can improve cardiovascular fitness and help the client tolerate periods of hypoxia better, perhaps reducing the risk of heart attack. Most exercise has little effect on respiratory muscle strength, and these clients can't tolerate the type of exercise necessary to do this. Exercise won't reduce the number of acute attacks. In some instances, exercise may be contraindicated, and the client should check with his physician before starting any exercise program.

An elderly client with pneumonia may appear with which of the following symptoms first? a. Altered mental status and dehydration b. fever and chills c. Hemoptysis and dyspnea d. Pleuretic chest pain and cough

ANSWER A. Fever, chills, hemoptysis, dyspnea, cough, and pleuric chest pain are the common symptoms of pneumonia, but elderly clients may first appear with only an altered mental status and dehydration due to a blunted immune response

20. Emergency treatment of a client in status asthmaticus includes which of the following medications? a. Inhaled beta-adrenergic agents b. Inhaled corticosteroids c. I.V. beta-adrenergic agents d. Oral corticosteroids

ANSWER A. Inhaled beta-adrenergic agents help promote bronchodilation, which improves oxygenation. I.V. beta-adrenergic agents can be used but have to be monitored because of their greater systemic effects. They're typically used when the inhaled beta-adrenergic agents don't work. Corticosteriods are slow-acting, so their use won't reduce hypoxia in the acute phase.

14. Clients with chronic obstructive bronchitis are given diuretic therapy. Which of the following reasons best explains why? a. Reducing fluid volume reduces oxygen demand. b. Reducing fluid volume improves clients' mobility. c. Restricting fluid volume reduces sputum production. d. Reducing fluid volume improves respiratory function.

ANSWER A. Reducing fluid volume reduces the workload of the heart, which reduces oxygen demand and, in turn, reduces the respiratory rate. It may also reduce edema and improve mobility a little, but exercise tolerance will still be harder to clear airways. Reducing fluid volume won't improve respiratory function, but may improve oxygenation.

6. A client with acute asthma showing inspiratory and expiratory wheezes and a decreased expiratory volume should be treated with which of the following classes of medication right away? a. Beta-adrenergic blockers b. Bronchodilators c. Inhaled steroids d. Oral steroids

ANSWER B. Bronchodilators are the first line of treatment for asthma because bronchoconstriction is the cause of reduced airflow. Beta-adrenergic blockers aren't used to treat asthma and can cause bronchoconstriction. Inhaled or oral steroids may be given to reduce the inflammation but aren't used for emergency relief.

7. A 19-year-old comes into the emergency department with acute asthma. His respiratory rate is 44 breaths/minute, and he appears to be in acute respiratory distress. Which of the following actions should be taken first? a. Take a full medication history b. Give a bronchodilator by neubulizer c. Apply a cardiac monitor to the client d. Provide emotional support to the client.

ANSWER B. The client is having an acute asthma attack and needs to increase oxygen delivery to the lung and body. Nebulized bronchodilators open airways and increase the amount of oxygen delivered. First resolve the acute phase of the attack ad how to prevent attacks in the future. It may not be necessary to place the client on a cardiac monitor because he's only 19-years-old, unless he has a past medical history of cardiac problems.

9. The term "blue bloater" refers to which of the following conditions? a. Adult respiratory distress syndrome (ARDS) b. Asthma c. Chronic obstructive bronchitis d. Emphysema

ANSWER C. Clients with chronic obstructive bronchitis appear bloated; they have large barrel chests and peripheral edema, cyanotic nail beds and, at times, circumoral cyanosis. Clients with ARDS are acutely short of breath and frequently need intubation for mechanical ventilation and large amounts of oxygen. Clients with asthma don't exhibit characteristics of chronic disease, and clients with emphysema appear pink and cachectic (a state of ill health, malnutrition, and wasting).

4. Which of the following assessment findings would help confirm a diagnosis of asthma in a client suspected of having the disorder? a. Circumoral cyanosis b. Increased forced expiratory volume c. Inspiratory and expiratory wheezing d. Normal breath sounds

ANSWER C. Inspiratory and expiratory wheezes are typical findings in asthma. Circumoral cyanosis may be present in extreme cases of respiratory distress. The nurse would expect the client to have a decreased forced expiratory volume because asthma is an obstructive pulmonary disease. Breath sounds will be "tight" sounding or markedly decreased; they won't be normal.

5. Which of the following types of asthma involves an acute asthma attack brought on by an upper respiratory infection? a. Emotional b. Extrinsic c. Intrinsic d. Mediated

ANSWER C. Intrinsic asthma doesn't have an easily identifiable allergen and can be triggered by the common cold. Asthma caused be emotional reasons is considered to be in the extrinsic category. Extrinsic asthma is caused by dust, molds, and pets; easily identifiable allergens. Mediated asthma doesn't exist.

10. The term "pink puffer" refers to the client with which of the following conditions? a. ARDS b. Asthma c. Chronic obstructive bronchitis d. Emphysema

ANSWER D. Because of the large amount of energy it takes to breathe, clients with emphysema are usually cachectic. They're pink and usually breathe through pursed lips, hence the term "puffer". Clients with ARDS are usually acutely short of breath. Clients with asthma don't have any particular characteristics, and clients with chronic obstructive bronchitis are bloated and cyanotic in appearance.

12. It's highly recommended that clients with asthma, chronic bronchitis, and emphysema have Pneumovax and flu vaccinations for which of the following reasons? a. All clients are recommended to have these vaccines b. These vaccines produce bronchodilation and improve oxygenation. c. These vaccines help reduce the tachypnea these clients experience. d. Respiratory infections can cause severe hypoxia and possibly death in these clients.

ANSWER D. It's highly recommended that clients with respiratory disorders be given vaccines to protect against respiratory infection. Infections can cause these clients to need intubation and mechanical ventilation, and it may be difficult to wean these clients from the ventilator. The vaccines have no effect on bronchodilation or respiratory care.

17. Teaching for a client with chronic obstructive pulmonary disease (COPD) should include which of the following topics? a. How to have his wife learn to listen to his lungs with a stethoscope from Wal-Mart. b. How to increase his oxygen therapy. c. How to treat respiratory infections without going to the physician. d. How to recognize the signs of an impending respiratory infection.

ANSWER D. Respiratory infection in clients with a respiratory disorder can be fatal. It's important that the client understands how to recognize the signs and symptoms of an impending respiratory infection. It isn't appropriate for the wife to listen to his lung sounds, besides, you can't purchase stethoscopes from Wal-Mart. If the client has signs and symptoms of an infection, he should contact his physician at once.

2. Which of the following pathophysiological mechanisms that occurs in the lung parenchyma allows pneumonia to develop? a. Atelectasis b. Bronchiectasis c. Effusion d. Inflammation

ANSWER D. The most common feature of all types of pneumonia is an inflammatory pulmonary response to the offending organism or agent. Atelectasis and brochiectasis indicate a collapse of a portion of the airway that doesn't occur with pneumonia. An effusion is an accumulation of excess pleural fluid in the pleural space, which may be a secondary response to pneumonia.

11. A 66-year-old client has marked dyspnea at rest, is thin, and uses accessory muscles to breathe. He's tachypneic, with a prolonged expiratory phase. He has no cough. He leans forward with his arms braced on his knees to support his chest and shoulders for breathing. This client has symptoms of which of the following respiratory disorders? a. ARDS b. Asthma c. Chronic obstructive bronchitis d. Emphysema

ANSWER D. These are classic signs and symptoms of a client with emphysema. Clients with ARDS are acutely short of breath and require emergency care; those with asthma are also acutely short of breath during an attack and appear very frightened. Clients with chronic obstructive bronchitis are bloated and cyanotic in appearance.

19. Which of the following measures can reduce or prevent the incidence of atelectasis in a post-operative client? a. Chest physiotherapy b. Mechanical ventilation c. Reducing oxygen requirements d. Use of an incentive spirometer

ANSWER D. Using an incentive spirometer requires the client to take deep breaths and promotes lung expansion. Chest physiotherapy helps mobilize secretions but won't prevent atelectasis. Reducing oxygen requirements or placing someone on mechanical ventilation doesn't affect the development of atelectasis.

21. The nurse is caring for a client diagnosed with pneumonia. The nurse plans which of the following as the best time to take the client for a short walk? a) after the client eats lunch b) after the client has a brief nap c) after the client uses the metered-dose inhaler d) after recording oxygen saturation on the bedside flow sheet

C - The nurse should schedule activities for the client with pneumonia after the client has received respiratory treatments or medications. After the administration of bronchodilators (often administered by metered-dose inhaler), the client has the best oxygen exchange possible and would tolerate the activity best. Still, the nurse implements activity cautiously, so as not to increase the client's dyspnea. The client would become fatigued after eating; therefore, this is not a good time to ambulate the client. Although the client may be rested somewhat after a nap, from the options provided this is not the best time to ambulate. Option D is unrelated to the client's ability to tolerate ambulation.

A nurse reviews the arterial blood gas (ABG) values of a client admitted with pneumonia: pH, 7.51; PaCO2, 28 mm Hg; PaO2, 70 mm Hg; and HCO3-, 24 mEq/L. What do these values indicate? a. Metabolic alkalosis b. Metabolic acidosis c. Respiratory alkalosis d. Respiratory acidosi

C A client with pneumonia may hyperventilate in an effort to increase oxygen intake. Hyperventilation leads to excess carbon dioxide (CO2) loss, which causes alkalosis — indicated by this client's elevated pH value. With respiratory alkalosis, the kidneys' bicarbonate (HCO3-) response is delayed, so the client's HCO3- level remains normal. The below-normal value for the partial pressure of arterial carbon dioxide (PaCO2) indicates CO2 loss and signals a respiratory component. Because the HCO3- level is normal, this imbalance has no metabolic component. Therefore, the client is experiencing respiratory alkalosis.

A client in the emergency department reports that he has been vomiting excessively for the past 2 days. His arterial blood gas analysis shows a pH of 7.50, partial pressure of arterial carbon dioxide (PaCO2) of 43 mm Hg, partial pressure of arterial oxygen (PaO2) of 75 mm Hg, and bicarbonate (HCO3-) of 42 mEq/L. Based on these findings, the nurse documents that the client is experiencing which type of acid-base imbalance? a. Respiratory acidosis b. Respiratory alkalosis c. Metabolic alkalosis d. Metabolic acidosis

C A pH over 7.45 with a HCO3- level over 26 mEq/L indicates metabolic alkalosis. Metabolic alkalosis is always secondary to an underlying cause and is marked by decreased amounts of acid or increased amounts of base HCO3-. The client isn't experiencing respiratory alkalosis because the PaCO2 is normal. The client isn't experiencing respiratory or metabolic acidosis because the pH is greater than 7.35.

A physician orders regular insulin 10 units I.V. along with 50 ml of dextrose 50% for a client with acute renal failure. What electrolyte imbalance is this client most likely experiencing? a. Hyperglycemia b. Hypercalcemia c. Hyperkalemia d. Hypernatremia

C Administering regular insulin I.V. concomitantly with 50 ml of dextrose 50% helps shift potassium from the extracellular fluid into the cell, which normalizes serum potassium levels in the client with hyperkalemia. This combination doesn't help reverse the effects of hypercalcemia, hypernatremia, or hyperglycemia.

An older client is receiving postural drainage treatments but is unable to expel the secretions. The client is confused, and having difficulty following instructions. The best response by the nurse would be to: A. Frequently change the patient's position B. Administer humidified oxygen C. Suction out the secretions. D. Have the patient drink water

C Postural drainage loosens secretions, which patients usually expectorate when loosened. Since the client is confused, and cannot follow instructions, the nurse must suction the secretions to enhance effective breathing.

Which of these nursing actions included in the care of a mechanically ventilated patient with acute respiratory distress syndrome (ARDS) is most appropriate for the RN to delegate to an experienced LPN/LVN working in the intensive care unit? a. Placing the patient in the prone position b. Assessment of patient breath sounds c. Administration of enteral tube feedings d. Obtaining the pulmonary artery pressures

C Rationale: Administration of tube feedings is included in LPN/LVN education and scope of practice and can be safely delegated to an LPN/LVN who is experienced in caring for critically ill patients. Placing a patient who is on a ventilator in the prone position requires multiple staff and should be supervised by an RN. Assessment of breath sounds and obtaining pulmonary artery pressures require advanced assessment skills and should be done by the RN caring for a critically ill patient.

All the following medications are ordered for a mechanically ventilated patient with acute respiratory distress syndrome (ARDS) and acute renal failure. Which medication should the nurse discuss with the health care provider before administration? a. IV ranitidine (Zantac) 50 mg IV b. sucralfate (Carafate) 1 g per nasogastric tube c. IV gentamicin (Garamycin) 60 mg d. IV methylprednisolone (Solu-Medrol) 40 mg

C Rationale: Gentamicin, which is one of the aminoglycoside antibiotics, is potentially nephrotoxic, and the nurse should clarify the drug and dosage with the health care provider before administration. The other medications are appropriate for the patient with ARDS.

It will be most important for the nurse to check pulse oximetry for which of these patients? a. A patient with emphysema and a respiratory rate of 16 b. A patient with massive obesity who is refusing to get out of bed c. A patient with pneumonia who has just been admitted to the unit d. A patient who has just received morphine sulfate for postoperative pain

C Rationale: Hypoxemia and hypoxemic respiratory failure are caused by disorders that interfere with the transfer of oxygen into the blood, such as pneumonia. The other listed disorders are more likely to cause problems with hypercapnia because of ventilatory failure.

When performing an assessment on the client with emphysema, the nurse finds that the client has a barrel chest. The alteration in the client's chest is due to: A. Collapse of distal alveoli B. Hyperinflation of the lungs C. Long-term chronic hypoxia D. Use of accessory muscles

Answer B is correct. Clients with emphysema develop a barrel chest due to the trapping of air in the lungs, causing them to hyperinflate. Answers C and D are common in those with emphysema but do not cause the chest to become barrel shaped. Answer A does not occur in emphysema.

The morning weight for a client with emphysema indicates that the client has gained 5 pounds in less than a week, even though his oral intake has been modest. The client's weight gain may reflect which associated complication of COPD? A. Polycythemia B. Cor pulmonale C Left ventricular failure D. Compensated acidosis

Answer B is correct. Cor pulmonale, or right sided heart failure, is a possible complication of emphysema. Answers A and D do not cause weight gain, so they're incorrect. Answer C would be reflected in pulmonary edema, so it's incorrect.

The nurse notes that a client with COPD demonstrates more dyspnea in certain positions. Which position is most likely to alleviate the client's dyspnea? A. Lying supine with a single pillow B. Standing or sitting upright C. Side lying with the head elevated D. Lying with head slightly lowered

Answer B is correct. The client with chronic obstructive pulmonary disease has increased difficulty breathing when lying down. His respiratory effort is improved by standing or sitting upright or by having the bed in high Fowler's position. Answers A, C, and D do not alleviate the client's dyspnea; therefore they are incorrect.

18. The MOST effective nursing intervention to prevent atelectasis from developing in a post operative client is to: a. Maintain adequate hydration b. Assist client to turn, cough and deep breathe c. Ambulate client within 12 hours d. Splint incision

Answer B. Deep air excursion by turning, coughing, and deep breathing will expand the lungs and stimulate surfactant production. The nurse should instruct the client on how to splint the chest when coughing. Humidification, hydration and nutrition all play a part in preventing atelectasis following surgery.

12. When caring for a client with a post right thoracotomy who has undergone an upper lobectomy, the nurse focuses on pain management to promote: a. Relaxation and sleep b. Coughing and deep breathing c. Incisional healing d. Range of motion exercises

Answer B. The priority is postoperative respiratory toilet. This client will quickly develop profound atelectasis and eventually pneumonia without adequate gas exchange. This will only be achieved with the appropriate pain management.

16. A client is diagnosed with a spontaneous pneumothorax necessitating the insertion of a chest tube. What is the BEST explanation for the nurse to provide this client? a. "The tube will drain fluid from your chest." b. "The tube will remove excess air from your chest." c. "The tube controls the amount of air that enters your chest." d. "The tube will seal the hole in your lung."

Answer B. The purpose of the chest tube is to create negative pressure and remove the air that has accumulated in the pleural space.

When reviewing the chart of a client with long standing lung disease, the nurse should pay close attention to the results of which pulmonary function test? A.Residual volume B. Total lung capacity C. FEV1/FVC ratio D. Functional residual capacity

Answer C is correct. The FEV1/FVC ratio indicates disease progression. As COPD worsens, the ratio of FEV1 to FVC becomes smaller. Answers A and B reflect loss of elastic recoil due to narrowing and obstruction of the airway. Answer D is increased in clients with obstructive bronchitis.

The nurse is teaching the client the appropriate way to use a metered dose inhaler. Which action indicates the client needs additional teaching? A. The client takes a deep breath while depressing the inhaler. B. The client places the inhaler two fingers from the mouth. C. The client waits 15 seconds before using the inhaler a second time. D. The client exhales slowly using purse lipped breathing.

Answer C is correct. The client should wait 60 seconds before using the inhaler a second time. The client's wait time of 15 seconds indicates that the client needs further teaching. Answers A, B, and D indicate that the client understands the correct use of the inhaler.

13. The priority is postoperative respiratory toilet. This client will quickly develop profound atelectasis and eventually pneumonia without adequate gas exchange. This will only be achieved with the appropriate pain management. a. Pallor b. Increased temperature c. Dyspnea d. Involuntary muscle spasms

Answer C. Client's having the insertion of a central venous catheter are at risk for tension pneumothorax. Dyspnea, shortness of breath and chest pain are indications of this complication.

The physician has ordered O2 at 3 liters/minute via nasal cannula. O2 amounts greater than 3 liters / minute are contraindicated in the client with COPD because: A.Higher concentrations result in severe headache. B.Hypercapnic drive is necessary for breathing. C. Higher levels will be required later to raise the pO2. D. hypoxic drive is needed for breathing

Answer D is correct. In clients with COPD, respiratory effort is stimulated by hypoxemia. Answers A and C are incorrect because higher levels would rob the client of the drive to breathe. Answer B is an incorrect statement.

The nurse is caring for a client with a history of advanced chronic obstructive pulmonary disease (COPD). The client had conventional gallbladder surgery 2 days previously. Which intervention has priority for preventing respiratory complications? A. Incentive spirometry every 4 hours. B. Coughing and deep breathing four times daily. C. Getting the client out of bed 4 times daily as ordered by the physician. D. Giving oxygen at 4 L/minute according to the physician's order.

Correct answer: C Getting the client out of bed prevents pooling of secretions in the lungs and promotes better lung expansion. An incentive spirometer (a device that measures air movement into the lungs and encourages the client to breathe deeply), coughing, and deep breathing are important, but the client needs to perform these more frequently (every 1 to 2 hours) instead of every 4 hours or 4 times daily. Giving oxygen at 4 L/minute could decrease the client's respiratory drive.

A 32323232-year-old man has been admitted to the emergency department with complaints of chest pain, shortness of breath and a fever for the last 3333 days. Upon assessment, the healthcare provider notes the patient has a productive cough with pus-like sputum. What is the most appropriate conclusion a healthcare provider can infer from this data? Please choose from one of the following options. A. Chest pain related to pneumonitis B.Ineffective breathing related pleural effusion C. Decreased appetite related to pneumonia D. Impaired gas exchanged related to pulmonary abscess

D

Match the acid-base status of the following blood samples to the disorders in the drop down list. (PaCO2 values are in mm Hg and bicarbonate values in mmol/l). pH 7.57, PaCO2 22, HCO3- 17 A. Respiratory Acidosis, Partially Compensated B. Respiratory Alkalosis, Uncompensated C. Metabolic Acidosis, Partially Compensated D. Respiratory Alkalosis, Partially Compensated

D

pH 7.17, PaCO2 48, HCO3- 36 A. Respiratory Acidosis, Uncompensated B. Metabolic Acidosis, Partially Compensated C. Respiratory Alkalosis, Partially Compensated D. Respiratory Acidosis, Partially Compensated

D

pH 7.39, PaCO2 44, HCO3- 26 A. Metabolic Acidosis B. Respiratory Alkalosis C. Respiratory Acidosis D. Normal

D

pH 7.39, PaCO2 44, HCO3- 26 A. Respiratory Acidosis B. Metabolic Acidosis C. Respiratory Alkalosis D. Normal

D

pH 7.5, PaCO2 19, HCO3- 22 A. .Respiratory Alkalosis, Partially Compensated B. Metabolic Alkalosis, Partially Compensated C. Respiratory Acidosis, Uncompensated D. Respiratory Alkalosis, Uncompensated

D

A patient with chronic obstructive pulmonary disease (COPD) arrives in the emergency department complaining of acute respiratory distress. When monitoring the patient, which assessment by the nurse will be of most concern? a. The patient is sitting in the tripod position. b. The patient has bibasilar lung crackles. c. The patient's pulse oximetry indicates an O2 saturation of 91%. d. The patient's respiratory rate has decreased from 30 to 10/min.

D Rationale: A decrease in respiratory rate in a patient with respiratory distress suggests the onset of fatigue and a high risk for respiratory arrest; therefore, the nurse will need to take immediate action. Patients who are experiencing respiratory distress frequently sit in the tripod position because it decreases the work of breathing. Crackles in the lung bases may be the baseline for a patient with COPD. An oxygen saturation of 91% is common in patients with COPD and will provide adequate gas exchange and tissue oxygenation.

The RN has conducted discharge teaching w/ a pt dx with TB, who has been receiving medication for 1.5 wks. The RN determines that the pt has understood the information if the client makes which statement? a) I need to cont. drug therapy for 2 months. b) I can't shop at the mall for the next 6 months c) I can return to work if a sputum culture comes back neg. d) I should not be contagious after 2 to 3 wks of medication therapy

D Medication therapy is 6-12 months. Pt generally considered noncontagious after 2-3 wks of med therapy. Wear a mask if exposed to crowds until med is effective. Allowed to return to work after three neg sputum cultures.

The cyanosis that accompanies bacterial pneumonia is primarily caused by which of the following? A. Decreased cardiac output B. Pleural effusion C. Inadequate peripheral circulation D. Decreased oxygenation of the blood

D A client with pneumonia has less lung surface available for the diffusion of gases because of the inflammatory pulmonary response that creates lung exudate and results in reduced oxygenation of the blood. The client becomes cyanotic because blood is not adequately oxygenated in the lungs before it enters the peripheral circulation.

A client with primary TB infection can expect to develop which of the following conditions? A. Active TB within 2 weeks B. Active TB within 1 month C. A fever that requires hospitalization D. A positive skin test

D A primary TB infection occurs when the bacillus has successfully invaded the entire body after entering through the lungs. At this point, the bacilli are walled off and skin tests read positive. However, all but infants and immunosuppressed people will remain asymptomatic. The general population has a 10% risk of developing active TB over their lifetime, in many cases because of a break in the body's immune defenses. The active stage shows the classic symptoms of TB: fever, hemoptysis, and night sweats.

A client with shortness of breath has decreased to absent breath sounds on the right side, from the apex to the base. Which of the following conditions would best explain this? A. Acute asthma B. Chronic bronchitis C. Pneumonia D. Spontaneous pneumothorax

D A spontaneous pneumothorax occurs when the client's lung collapses, causing an acute decrease in the amount of functional lung used in oxygenation. The sudden collapse was the cause of his chest pain and shortness of breath. An asthma attack would show wheezing breath sounds, and bronchitis would have rhonchi. Pneumonia would have bronchial breath sounds over the area of consolidation.

A chest x-ray should a client's lungs to be clear. His Mantoux test is positive, with a 10mm if induration. His previous test was negative. These test results are possible because: A. He had TB in the past and no longer has it. B. He was successfully treated for TB, but skin tests always stay positive C. He's a "seroconverter", meaning the TB has gotten to his bloodstream D. He's a "tuberculin converter," which means he has been infected with TB since his last skin test

D A tuberculin converter's skin test will be positive, meaning he has been exposed to an infected with TB and now has a cell-mediated immune response to the skin test. The client's blood and x-ray results may stay negative. It doesn't mean the infection has advanced to the active stage. Because his x-ray is negative, he should be monitored every 6 months to see if he develops changes in his x-ray or pulmonary examination. Being a seroconverter doesn't mean the TB has gotten into his bloodstream; it means it can be detected by a blood test.

A client with a positive skin test for TB isn't showing signs of active disease. To help prevent the development of active TB, the client should be treated with isoniazid, 300 mg daily, for how long? A. 10 to 14 days B. 2 to 4 weeks C. 3 to 6 months D. 9 to 12 month

D Because of the increased incidence of resistant strains of TB, the disease must be treated for up to 24 months in some cases, but treatment typically lasts for 9-12 months. Isoniazid is the most common medication used for the treatment of TB, but other antibiotics are added to the regimen to obtain the best results.

A client is diagnosed with active TB and started on triple antibiotic therapy. What signs and symptoms would the client show if therapy is inadequate? A. Decreased shortness of breath B. Improved chest x-ray C. Nonproductive cough D. Positive acid-fast bacilli in a sputum sample after 2 months of treatment

D Continuing to have acid-fast bacilli in the sputum after 2 months indicated continued infection.

When assessing a client for obstructive sleep apnea (OSA), the nurse understands the most common symptom is: A. Headache B. Early awakening C. Impaired reasoning D. Excessive daytime sleepiness

D Excessive daytime sleepiness is the most common complaint of people with OSA. Persons with severe OSA may report taking daytime naps and experiencing a disruption in their daily activities because of sleepiness.

Which of the following statements best explains how opening up collapsed alveoli improves oxygenation? A. Alveoli need oxygen to live B. Alveoli have no effect on oxygenation C. Collapsed alveoli increase oxygen demand D. Gaseous exchange occurs in the alveolar membrane

D Gaseous exchange occurs in the alveolar membrane, so if the alveoli collapse, no exchange occurs, Collapsed alveoli receive oxygen, as well as other nutrients, from the bloodstream. Collapsed alveoli have no effect on oxygen demand, though by decreasing the surface area available for gas exchange, they decrease oxygenation of the blood.

The nurse should include which of the following instructions when developing a teaching plan for clients receiving INH and rifampin for treatment for TB? A. Take the medication with antacids B. Double the dosage if a drug dose is forgotten C. Increase intake of dairy products D. Limit alcohol intake

D INH and rifampin are hepatotoxic drugs. Clients should be warned to limit intake of alcohol during drug therapy. Both drugs should be taken on an empty stomach. If antacids are needed for GI distress, they should be taken 1 hour before or 2 hours after these drugs are administered. Clients should not double the dosage of these drugs because of their potential toxicity. Clients taking INH should avoid foods that are rich in tyramine, such as cheese and dairy products, or they may develop hypertension.

A client has been treated with antibiotic therapy for right lower-lobe pneumonia for 10 days and will be discharged today. Which of the following physical findings would lead the nurse to believe it is appropriate to discharge this client? A. Continued dyspnea B. Fever of 102*F C. Respiratory rate of 32 breaths/minute D. Vesicular breath sounds in right base

D If the client still has pneumonia, the breath sounds in the right base will be bronchial, not the normal vesicular breath sounds. If the client still has dyspnea, fever, and increased respiratory rate, he should be examined by the physician before discharge because he may have another source of infection or still have pneumonia.

In a patient undergoing surgery, it was vital to aspirate the contents of the upper gastrointestinal tract. After the operation, the following values were acquired from an arterial blood sample: pH 7.55, PCO2 52 mm Hg and HCO3- 40 mmol/l. What is the underlying disorder? A. Respiratory Acidosis B. Respiratory Alkalosis C. Metabolic Acidosis D. Metabolic Alkalosis

D NGT suctioning, vomiting, hypokalemia and overdosage of NaHCO3 are considered risk factors of metabolic alkalosis.

A 79-year-old client is admitted with pneumonia. Which nursing diagnosis should take priority? A. Acute pain related to lung expansion secondary to lung infection B. Risk for imbalanced fluid volume related to increased insensible fluid losses secondary to fever C. Anxiety related to dyspnea and chest pain D. Ineffective airway clearance related to retained secretions

D Pneumonia is an acute infection of the lung parenchyma. The inflammatory reaction may cause an outpouring of exudate into the alveolar spaces, leading to an ineffective airway clearance related to retained secretions.

A diagnosis of pneumonia is typically achieved by which of the following diagnostic tests? A. ABG analysis B. Chest x-ray C. Blood cultures D. Sputum culture and sensitivity

D Sputum C & S is the best way to identify the organism causing the pneumonia. Chest x-ray will show the area of lung consolidation. ABG analysis will determine the extent of hypoxia present due to the pneumonia, and blood cultures will help determine if the infection is systemic.

A nurse is caring for a client diagnosed with TB. Which assessment, if made by the nurse, would not be consistent with the usual clinical presentation of TB and may indicate the development of a concurrent problem? A. Nonproductive or productive cough B. Anorexia and weight loss C. Chills and night sweats D. High-grade fever

D The client with TB usually experiences cough (non-productive or productive), fatigue, anorexia, weight loss, dyspnea, hemoptysis, chest discomfort or pain, chills and sweats (which may occur at night), and a low-grade fever.

A male client undergoes a laryngectomy to treat laryngeal cancer. When teaching the client how to care for the neck stoma, the nurse should include which instruction? A. "Keep the stoma uncovered." B. "Keep the stoma dry." C. "Have a family member perform stoma care initially until you get used to the procedure." D. "Keep the stoma moist."

D The nurse should instruct the client to keep the stoma moist, such as by applying a thin layer of petroleum jelly around the edges, because a dry stoma may become irritated. The nurse should recommend placing a stoma bib over the stoma to filter and warm air before it enters the stoma. The client should begin performing stoma care without assistance as soon as possible to gain independence in self-care activities.

Which of the following best describes pleural effusion? A. The collapse of alveoli B. The collapse of bronchiole C. The fluid in the alveolar space D. The accumulation of fluid between the linings of the pleural space

D The pleural fluid normally seeps continually into the pleural space from the capillaries lining the parietal pleura and is reabsorbed by the visceral pleural capillaries and lymphatics. Any condition that interferes with either the secretion or drainage of this fluid will lead to a pleural effusion.

Carl, an elementary student, was rushed to the hospital due to vomiting and a decreased level of consciousness. The patient displays slow and deep (Kussmaul breathing), and he is lethargic and irritable in response to stimulation. He appears to be dehydrated—his eyes are sunken and mucous membranes are dry—and he has a two week history of polydipsia, polyuria, and weight loss. Measurement of arterial blood gas shows pH 7.0, PaO2 90 mm Hg, PaCO2 23 mm Hg, and HCO3 12 mmol/L; other results are Na+ 126 mmol/L, K+ 5 mmol/L, and Cl- 95 mmol/L. What is your assessment? A. Respiratory Acidosis, Uncompensated B. Respiratory Acidosis, Partially Compensated C. Metabolic Alkalosis, Uncompensated D. Metabolic Acidosis, Partially Compensated

D The student was diagnosed having diabetes mellitus. The results show that he has metabolic acidosis (low HCO3 -) with respiratory compensation (low CO2).

Dave, a 6-year-old boy, was rushed to the hospital following her mother's complaint that her son has been vomiting, nauseated and has overall weakness. After series of tests, the nurse notes the laboratory results: potassium: 2.9 mEq. Which primary acid-base imbalance is this boy at risk for if medical intervention is not carried out? A. Respiratory Acidosis B.Respiratory Alkalosis C. Metabolic Acidosis D. Metabolic Alkalosis

D Vomiting, hypokalemia, overdosage of NaHCO3 and NGT suctioning are considered risk factors of metabolic alkalosis.

38. In teaching the patient with COPD about the need for physical exercise, the nurse informs the patient that A. all patients with COPD should be able to increase walking gradually up to 20 min/day. B. a bronchodilator inhaler should be used to relieve exercise-induced dyspnea immediately after exercise. C. shortness of breath is expected during exercise but should return to baseline within 5 minutes after the exercise. D. monitoring the heart rate before and after exercise is the best way to determine how much exercise can be tolerated.

C. shortness of breath is expected during exercise but should return to baseline within 5 minutes after the exercise.Shortness of breath usually increases during exercise, but the activity is not being overdone if breathing returns to baseline within 5 minutes after stopping. Bronchodilators can be administered 10 minutes before exercise but should not be administered for at least 5 minutes after activity to allow recovery. Patients are encouraged to walk 15 to 20 minutes a day with gradual increases, but actual patterns will depend on patient tolerance. Dyspnea most frequently limits exercise and is a better indication of exercise tolerance than is heart rate in the patient with COPD.

14. Upon entering the room of a patient who has just returned from surgery for total laryngectomy and radical neck dissection, a nurse should recognize a need for intervention when finding A. a gastrostomy tube that is clamped. B. the patient coughing blood-tinged secretions from the tracheostomy. C. the patient positioned in a lateral position with the head of the bed flat. D. 200 ml of serosanguineous drainage in the patient's portable drainage device.

C. the patient positioned in a lateral position with the head of the bed flat. After total laryngectomy and radical neck dissection, a patient should be placed in a semi-Fowler's position to decrease edema and limit tension on the suture line.

17. A hospitalized client is dyspneic and has been diagnosed with left tension pneumothorax by chest X-ray after insertion of a central venous catheter. Which of the following observed by the nurse indicates that the pneumothorax is rapidly worsening? a) hypertension b) pain with respiration c) tracheal deviation to the left d) tracheal deviation to the right

D - A tension pneumothorax is characterized by distended neck veins, displaced point of maximal impulse (PMI), tracheal deviation to the unaffected side, asymmetry of the thorax, decreased to absent breath sounds on the affected side, and worsening cyanosis. The client could have pain with respiration. The increased intrathoracic pressure would cause the blood pressure to fall, not rise.

19. A client is suspected of having a pleural effusion. The nurse assesses the client for which typical manifestations of this respiratory problem? a) dyspnea at rest and moist, productive cough b) dyspnea at rest and dry, nonproductive cough c) dyspnea on exertion and moist, productive cough d) dyspnea on exertion and dry, nonproductive cough

D - Typical assessment findings in the client with a pleural effusion include dyspnea, which usually occurs with exertion, and a dry, nonproductive cough. The cough is caused by bronchial irritation and possible mediastinal shift.

High plasma PaCO2 A. Metabolic Acidosis B. Respiratory Alkalosis C. Metabolic Alkalosis D. Respiratory Acidosis

D An excess of carbon dioxide (hypercapnia) can cause carbon dioxide narcosis. In this condition, carbon dioxide levels are so high that they no longer stimulate respirations but depress them.

A nurse is reviewing the laboratory test results of a client with renal disease. Which of the following would the nurse expect to find? a. Decreased blood urea nitrogen (BUN) b. Decreased potassium c. Increased serum albumin d. Increased serum creatinine

D In clients with renal disease, the serum creatinine level would be increased. The BUN also would be increased, serum albumin would be decreased, and potassium would likely be increased.

A nurse is instructing a hospitalized client with a diagnosis of emphysema about measures that will enhance the effectiveness of breathing during dyspneic periods. Which position should the nurse instruct the client to assume? A. sitting up in bed B. side-lying in bed C. sitting in a recliner chair D. sitting on the side of the bed and leaning on an overbed table

D Positions that will assist the client with emphysema with breathing include sitting up and leaning on an overbed table, sitting up and resting the elbows on the knees, and standing and leaning against the wall

11. The nurse evaluates that discharge teaching for a patient hospitalized with pneumonia has been most effective when the patient states which of the following measures to prevent a relapse? A. "I will increase my food intake to 2400 calories a day to keep my immune system well." B. "I must use home oxygen therapy for 3 months and then will have a chest x-ray to reevaluate." C. "I will seek immediate medical treatment for any upper respiratory infections." D. "I should continue to do deep-breathing and coughing exercises for at least 6 weeks."

D. "I should continue to do deep-breathing and coughing exercises for at least 6 weeks." It is important for the patient to continue with coughing and deep breathing exercises for 6 to 8 weeks until all of the infection has cleared from the lungs. A patient should seek medical treatment for upper respiratory infections that persist for more than 7 days. Increased fluid intake, not caloric intake, is required to liquefy secretions. Home O2 is not a requirement unless the patient's oxygenation saturation is below normal.

27. A patient with COPD is receiving oxygen at 2 L/min. While in the supine position for a bath, the patient complains of shortness of breath. What is the most appropriate first nursing action? A. Increase the flow of oxygen. B. Perform tracheal suctioning. C. Report this to the physician. D. Assist the patient to Fowler's position.

D. Assist the patient to Fowler's position. Breathing is easier in Fowler's position because it permits greater expansion of the chest cavity. If repositioning does not improve the situation, then oxygenation and physician reporting might be appropriate. The patient would not benefit from tracheal suctioning.

22. Respiratory acidosis is at highest risk in a patient with A. hypokalemia. B. pulmonary fibrosis. C. salicylate overdose. D. COPD.

D. COPD. Chronic respiratory acidosis is most commonly caused by COPD. Pulmonary fibrosis, hypokalemia, and salicylate overdose do not predispose a patient to respiratory acidosis. Hypokalemia can lead to cardiac dysrhythmias. Salicylate overdose results in central nervous system changes, and pulmonary fibrosis can result in respiratory arrest.

22. The nurse is assigned to care for a patient who has anxiety and an exacerbation of asthma. Which of the following is the primary reason for the nurse to carefully inspect the chest wall of this patient? A. Observe for signs of diaphoresis B. Allow time to calm the patient C. Monitor the patient for bilateral chest expansion D. Evaluate the use of intercostal muscles

D. Evaluate the use of intercostal muscles The nurse physically inspects the chest wall to evaluate the use of intercostal (accessory) muscles, which gives an indication of the degree of respiratory distress experienced by the patient.

41. When planning patient teaching about emphysema, the nurse understands that the symptoms of emphysema are caused by which of the following? A. Hypertrophy and hyperplasia of goblet cells in the bronchi B. Collapse and hypoventilation of the terminal respiratory unit C. An overproduction of the antiprotease alpha1-antitrypsin D. Hyperinflation of alveoli and destruction of alveolar walls

D. Hyperinflation of alveoli and destruction of alveolar walls In emphysema, there are structural changes that include hyperinflation of alveoli, destruction of alveolar walls, destruction of alveolar capillary walls, narrowing of small airways, and loss of lung elasticity.

5. A nurse is performing assessment for a patient diagnosed with chronic obstructive pulmonary disease (COPD). Which of the following findings should the nurse expect to observe? A. Nonproductive cough B. Prolonged inspiration C. Vesicular breath sounds D. Increased anterior-posterior chest diameter

D. Increased anterior-posterior chest diameter An increased anterior-posterior diameter is a compensatory mechanism experienced by patients with COPD and is caused by air-trapping. Patients with COPD have a productive cough, often expectorating copious amounts of sputum. Because of air-trapping, patients with COPD experience a prolonged expiration because the rate of gas on exhalation takes longer to escape. Chest auscultation for patients with COPD often reveals wheezing, crackles, and other adventitious breath sounds.

38. The nurse is evaluating whether a patient understands how to safely determine whether a metered dose inhaler is empty. The nurse interprets that the patient understands this important information to prevent medication underdosing when the patient describes which method to check the inhaler? A. Place it in water to see if it floats. B. Shake the canister while holding it next to the ear. C. Check the indicator line on the side of the canister. D. Keep track of the number of inhalations used.

D. Keep track of the number of inhalations used. It is no longer appropriate to see if a canister floats in water or not as research has demonstrated this is not accurate. The best method to determine when to replace an inhaler is by knowing the maximum puffs available per MDI and then replacing when those inhalations have been used.

A patient is diagnosed with a large pulmonary embolism. When explaining to the patient what has happened to cause respiratory failure, which information will the nurse include? a. "Oxygen transfer into your blood is slow because of thick membranes between the small air sacs and the lung circulation." b. "Thick secretions in your small airways are blocking air from moving into the small air sacs in your lungs." c. "Large areas of your lungs are getting good blood flow but are not receiving enough air to fill the small air sacs." d. "Blood flow though some areas of your lungs is decreased even though you are taking adequate breaths."

D Rationale: A pulmonary embolus limits blood flow but does not affect ventilation, leading to a ventilation-perfusion mismatch. The response beginning, "Oxygen transfer into your blood is slow because of thick membranes" describes a diffusion problem. The remaining two responses describe ventilation-perfusion mismatch with adequate blood flow but poor ventilation.

Which statement by the nurse when explaining the purpose of positive end-expiratory pressure (PEEP) to the family members of a patient with ARDS is correct? a. "PEEP will prevent fibrosis of the lung from occurring." b. "PEEP will push more air into the lungs during inhalation." c. "PEEP allows the ventilator to deliver 100% oxygen to the lungs." d. "PEEP prevents the lung air sacs from collapsing during exhalation."

D Rationale: By preventing alveolar collapse during expiration, PEEP improves gas exchange and oxygenation. PEEP will not prevent the fibrotic changes that occur with ARDS, push more air into the lungs, or change the fraction of inspired oxygen (FIO2) delivered to the patient.

While caring for a patient who has been admitted with a pulmonary embolism, the nurse notes a change in the patient's arterial oxyhemoglobin saturation (SpO2) from 94% to 88%. The nurse will a. assist the patient to cough and deep-breathe. b. help the patient to sit in a more upright position. c. suction the patient's oropharynx. d. increase the oxygen flow rate.

D Rationale: Increasing oxygen flow rate will usually improve oxygen saturation in patients with ventilation-perfusion mismatch, as occurs with pulmonary embolism. Because the problem is with perfusion, actions that improve ventilation, such as deep-breathing and coughing, sitting upright, and suctioning, are not likely to improve oxygenation.

After prolonged cardiopulmonary bypass, a patient develops increasing shortness of breath and hypoxemia. To determine whether the patient has acute respiratory distress syndrome (ARDS) or pulmonary edema caused by left ventricular failure, the nurse will anticipate assisting with a. positioning the patient for a chest radiograph. b. drawing blood for arterial blood gases. c. obtaining a ventilation-perfusion scan. d. inserting a pulmonary artery catheter.

D Rationale: Pulmonary artery wedge pressure will remain at normal levels in the patient with ARDS because the fluid in the alveoli is caused by increased permeability of the alveolar-capillary membrane rather than by the backup of fluid from the lungs (as occurs in cardiogenic pulmonary edema). The other tests will not help in differentiating cardiogenic from noncardiogenic pulmonary edema.

A patient in acute respiratory failure as a complication of COPD has a PaCO2 of 65 mm Hg, rhonchi audible in the right lung, and marked fatigue with a weak cough. The nurse will plan to a. allow the patient to rest to help conserve energy. b. arrange for a humidifier to be placed in the patient's room. c. position the patient on the right side with the head of the bed elevated. d. assist the patient with augmented coughing to remove respiratory secretions.

D Rationale: The patient's assessment indicates that assisted coughing is needed to help remove secretions, which will improve PaCO2 and will also help to correct fatigue. If the patient is allowed to rest, the PaCO2 will increase. Humidification may help loosen secretions, but the weak cough effort will prevent the secretions from being cleared. The patient should be positioned with the good lung down to improve gas exchange.

The nurse is caring for a patient who was hospitalized 2 days earlier with aspiration pneumonia. Which assessment information is most important to communicate to the health care provider? a. The patient has a cough that is productive of blood-tinged sputum. b. The patient has scattered crackles throughout the posterior lung bases. c. The patient's temperature is 101.5° F after 2 days of IV antibiotic therapy. d. The patient's SpO2 has dropped to 90%, although the O2 flow rate has been increased.

D Rationale: The patient's dropping SpO2 despite having an increase in FIO2 indicates the possibility of acute respiratory distress syndrome (ARDS). The patient's blood-tinged sputum and scattered crackles are not unusual in a patient with pneumonia, although they do require continued monitoring. The continued temperature elevation indicates a possible need to change antibiotics, but this is not as urgent a concern as the progression toward hypoxemia despite an increase in O2 flow rate.

A client with a suspected overdose of an unknown drug is admitted to the emergency department. Arterial blood gas values indicate respiratory acidosis. What should the nurse do first? a. Prepare for gastric lavage. b. Monitor the client's heart rhythm. c. Obtain a urine specimen for drug screening. d. Prepare to assist with ventilation

D Respiratory acidosis is associated with hypoventilation; in this client, hypoventilation suggests intake of a drug that has suppressed the brain's respiratory center. Therefore, the nurse should assume the client has respiratory depression and should prepare to assist with ventilation. After the client's respiratory function has been stabilized, the nurse can safely monitor the heart rhythm, prepare for gastric lavage, and obtain a urine specimen for drug screening.

A client develops decreased renal function and requires a change in antibiotic dosage. On which factor should the physician base the dosage change? a. Therapeutic index b. GI absorption rate c. Liver function studies d. Creatinine clearance

D The physician should base changes to antibiotic dosages on creatinine clearance test results, which gauge the kidney's glomerular filtration rate; this factor is important because most drugs are excreted at least partially by the kidneys. The GI absorption rate, therapeutic index, and liver function studies don't help determine dosage change in a client with decreased renal function.

The nurse instructs a client to use the pursed-lip method of breathing and the client asks the nurse about the purpose of this type of breathing. The nurse responds, knowing that the primary purpose is to promote which outcome? A. Promote oxygen intake B. Strengthen the diaphragm C. Strengthen intercostal muscles D. Promote carbon dioxide elimination

D This type of breathing allows better expiration by increasing airway pressure that keeps air passages open during exhalation. Pursed-lip breathing facilitates maximal expiration for clients with obstructive lung disease.

You are caring for a client with severe hypokalemia. The physician has ordered IV potassium to be administered at 10 mEq/hr. The client complains of burning along their vein. What should you do? a. Change the electrolyte. b. Switch to an oral formulation. c. Increase the speed of transfusion. d. Dilute the infusion

D Treatment of severe hypokalemia requires treatment with IV infusion of potassium. Clients may experience burning along the vein with IV infusion of potassium in proportion to the infusion's concentration. If the client can tolerate the fluid, consult with the physician about diluting the potassium in a larger volume of IV solution. Oral potassium may not be enough in severe cases hypokalemia. Hypokalemia requires treatment with potassium and not any other electrolyte.

44. When caring for a patient with COPD, the nurse identifies a nursing diagnosis of imbalanced nutrition less than body requirements after noting a weight loss of 30 lb. Which of the following would be an appropriate intervention to add to the plan of care for this patient? A. Teach the patient to use frozen meals at home that can be microwaved. B. Provide a high-calorie, high-carbohydrate, nonirritating, frequent feeding diet. C. Order fruits and fruit juices to be offered between meals. D. Order a high-calorie, high-protein diet with six small meals a day.

D. Order a high-calorie, high-protein diet with six small meals a day.Because the patient with COPD needs to use greater energy to breathe, there is often decreased oral intake because of dyspnea. A full stomach also impairs the ability of the diaphragm to descend during inspiration, interfering with the work of breathing. Finally, the metabolism of a high carbohydrate diet yields large amounts of CO2, which may lead to acidosis in patients with pulmonary disease. For these reasons, the patient with emphysema should take in a high-calorie, high-protein diet, eating six small meals per day.

33. The nurse determines that the patient is not experiencing adverse effects of albuterol (Proventil) after noting which of the following patient vital signs? A. Oxygen saturation 96% B. Respiratory rate of 18 C. Temperature of 98.4° F D. Pulse rate of 76

D. Pulse rate of 76 Albuterol is a β2-agonist that can sometimes cause adverse cardiovascular effects. These would include tachycardia and angina. A pulse rate of 76 indicates that the patient did not experience tachycardia as an adverse effect.

36. The nurse is assisting a patient to learn self-administration of beclomethasone two puffs inhalation q6hr. The nurse explains that the best way to prevent oral infection while taking this medication is to do which of the following as part of the self-administration techniques? A. Chew a hard candy before the first puff of medication. B. Ask for a breath mint following the second puff of medication. C. Rinse the mouth with water before each puff of medication. D. Rinse the mouth with water following the second puff of medication.

D. Rinse the mouth with water following the second puff of medication. The patient should rinse the mouth with water following the second puff of medication to reduce the risk of fungal overgrowth and oral infection.

4. After a posterior nasal pack is inserted by a physician, the patient is very anxious and states, "I don't feel like I'm breathing right." The immediate intervention the nurse should initiate is to A. monitor ABGs. B. reassure the patient that this is normal discomfort. C. cut the pack strings and pull the packing out with a hemostat. D. direct a flashlight into the patient's mouth and inspect the oral cavity.

D. direct a flashlight into the patient's mouth and inspect the oral cavity. The nurse should inspect the oral cavity for the presence of blood, soft palate necrosis, and proper placement of the posterior plug. If the posterior plug is visible, the physician should be notified for readjustment of the packing. Reassurance, cutting the strings, and ABGs are not top priority interventions. The nurse needs further data before intervening.

10. Anticoagulant therapy is used in the treatment of thromboembolic disease because anticoagulants can A. dissolve the thrombi. B. decrease blood viscosity. C. prevent absorption of vitamin K. D. inhibit the synthesis of clotting factors.

D. inhibit the synthesis of clotting factors. Anticoagulant therapy is based on the premise that the initiation or extension of thrombi can be prevented by inhibiting the synthesis of clotting factors or by accelerating their inactivation. The anticoagulants heparin and warfarin do not induce thrombolysis but effectively prevent clot extension.

31. A patient's ABGs include a PaO2 of 88 mm Hg and a PaCO2 of 38 mm Hg and mixed venous blood gases include a PvO2 of 40 mm Hg and PvCO2 of 46 mm Hg. These findings indicate that the patient has A. impaired cardiac output. B. unstable hemodynamics. C. inadequate delivery of oxygen to the tissues. D. normal capillary oxygen-carbon dioxide exchange.

D. normal capillary oxygen-carbon dioxide exchange. Normal venous blood gas values reflect the normal uptake of oxygen from arterial blood and the release of carbon dioxide from cells into the blood, resulting in a much lower PaO2 and an increased PaCO2. The pH is also decreased in mixed venous blood gases because of the higher PvCO2. Normal mixed venous blood gases also have much lower PvO2 and SvO2 than arterial blood bases. Mixed venous blood gases are used when patients are hemodynamically unstable to evaluate the amount of oxygen delivered to the tissue and the amount of oxygen consumed by the tissues.

7. The most appropriate position to assist a patient with chronic obstructive pulmonary disease (COPD) who is having difficulty breathing would be a A. high Fowler's position without a pillow behind the head. B. semi-Fowler's position with a single pillow behind the head. C. right side-lying position with the head of the bed at 45 degrees' elevation. D. sitting upright and forward position with arms supported on an over-the-bed table.

D. sitting upright and forward position with arms supported on an over-the-bed table.Sitting upright and leaning forward with arms supported on an over-the-bed table would be of most help to this patient, because it allows for expansion of the thoracic cage in all four directions (front, back, and two sides).

15. When administering oxygen to a patient with COPD with the potential for carbon dioxide narcosis, the nurse should A. never administer oxygen at a rate of more than 2 L/min. B. monitor the patient's use of oxygen to detect oxygen dependency. C. monitor the patient for symptoms of oxygen toxicity, such as paresthesias. D. use ABGs as a guide to determine what FIO2 level meets the patient's needs.

D. use ABGs as a guide to determine what FIO2 level meets the patient's needs. It is critical to start oxygen at low flow rates and then use ABGs as a guide to determine what FIO2 level is sufficient and can be tolerated.

A pt who is HIV-pos has had a TB skin test. The RN notes a 7mm area of induration at the site of the skin test and interprets the result as which finding? Positive Negative Inconclusive Need for repeat testing

Positive The pt w/ HIV is considered to have a pos skin test with area of induration larger than 5mm. A pt w/o HIV is positive with an induration larger than 10mm. HIV makes a pt immunosuppressed.

A client's ABG analysis reveals a pH of 7.18, PaCO2 of 72 mm Hg, PaO2 of 77 mm Hg, and HCO3- of 24 mEq/L. What do these values indicate?

Respiratory acidosis

The RN performs an admission assessment on a client w/ a dx of TB. The RN should check the results of which diagnostic test that will confirm this dx? Chest x-ray Bronchoscopy Sputum Culture TB skin test

Sputum culture TB is definitively diagnosed through culture and isolation of Mycobacterium tuberculosis.

Clients with chronic illnesses are more likely to get pneumonia when which of the following situations is present? 1. Dehydration 2. Group living 3. Malnutrition 4. Severe periodontal disease

2. Clients with chronic illnesses generally have poor immune systems. Often, residing in group living situations increases the chance of disease transmission.

A 7-year-old client is brought to the E.R. He's tachypneic and afebrile and has a respiratory rate of 36 breaths/minute and a nonproductive cough. He recently had a cold. From his history, the client may have which of the following? a. Acute asthma b. Bronchial pneumonia c. Chronic obstructive pulmonary disease (COPD) d. Emphysema

3. ANSWER A. Based on the client's history and symptoms, acute asthma is the most likely diagnosis. He's unlikely to have bronchial pneumonia without a productive cough and fever and he's too young to have developed COPD or emphysema.

A person was admitted to hospital in a coma. Analysis of the arterial blood gave the following values: PCO2 16 mm Hg, HCO3- 5 mmol/l and pH 7.1. What is the underlying acid-base disorder? a) Metabolic acidosis b) Respiratory alkalosis c) Metabolic alkalosis d) Respiratory acidosis

A The pH is below normal so there is an acidosis. The PCO2 and HCO3- are both below normal, so the underlying condition is a metabolic acidosis.

The nurse is checking laboratory values on a patient who has crackling rales in the lower lobes, 2+ pitting edema, and dyspnea with minimal exertion. Which of the following laboratory values does the nurse expect to be abnormal? A. Potassium. B. B-type natriuretic peptide (BNP). C. C-reactive protein (CRP). D. Platelets.

Correct answer: B The client's symptoms suggest heart failure. BNP is a neurohormone that is released from the ventricles when the ventricles experience increased pressure and stretch, such as in heart failure. A BNP level greater than 51 pg/mL is often associated with mild heart failure; and, as the BNP level increases, the severity of heart failure increases. Potassium levels are not affected by heart failure. CRP is an indicator of inflammation. It is used to help predict the risk of coronary artery disease. There is no indication that the client has an increased CRP. There is no indication that the client is experiencing bleeding or clotting abnormalities, such as those seen with an abnormal platelet count.

pH 7.6, PaCO2 53, HCO3- 38 A. Metabolic Alkalosis, Partially Compensated B. Metabolic Alkalosis, Fully Compensated C. Respiratory Acidosis, Partially Compensated D. Respiratory Alkalosis, Fully Compensated

A

Which of the following pathophysiological mechanisms that occurs in the lung parenchyma allows pneumonia to develop? 1. Atelectasis 2. Bronchiectasis 3. Effusion 4. Inflammation

4. The common feature of all type of pneumonia is an inflammatory pulmonary response to the offending organism or agent. Atelectasis and bronchiecrasis indicate a collapse of a portion of the airway that doesn't occur in pneumonia. An effusion is an accumulation of excess pleural fluid in the pleural space, which may be a secondary response to pneumonia.

To evaluate both oxygenation and ventilation in a patient with acute respiratory failure, the nurse uses the findings revealed with a. arterial blood gas (ABG) analysis. b. hemodynamic monitoring. c. chest x-rays. d. pulse oximetry.

A Rationale: ABG analysis is useful because it provides information about both oxygenation and ventilation and assists with determining possible etiologies and appropriate treatment. The other tests may also provide useful information about patient status but will not indicate whether the patient has hypoxemia, hypercapnia, or both.

The nurse expects which of the following assessment findings in the client in the diuretic phase of acute renal failure? a. Dehydration b. Crackles c. Hypertension d. Hyperkalemia

A The diuretic phase of acute renal failure is characterized by increased urine output, hypotension, and dehydration.

The RN is preparing a lsit of home care instructions for a client who has been hospitalized and treated for TB. Which instuctions should the RN include on the list? Select all that apply. a) Activities should be resumed gradually. b) Avoid contact with other individuals, except family members, for at least 6 months. c) A sputum culture is needed every 2 to 4 wks once medication therapy is initiated. d) Respiratory isolation is not necessary b/c family members already hace been exposed. e) Cover the mouth and nose when coughing or sneezing and put used tissues in plastic bags f) when one sputum culture is neg, the client is no longer considered infectious and usually can return to former employment

A, C, D, E -follow med regimen exactly - after 2-3 wks of therapy, it is unlikely to infect anyone - resume activities gradually - diet rich in iron, protein, and vit C for healing and preventing recurrence - when results of three sputum cultures are neg, then the pt is no longer considered infectious.

The nurse notes that a hospitalized patient has a positive reaction to the tuberculin skin test. Which action by the nurse is the priority? a. report the findings. b. document the findings. c. call employee health service d. call radiology for a chest x ray.

A. Reporting the findings is priority. The HCP will then order a chest x ray, and sputum culture, and patient will be placed of airborne precautions.

35. A patient is admitted to the hospital with fever, chills, a productive cough with rusty sputum, and pleuritic chest pain. Pneumococcal pneumonia is suspected. An appropriate nursing diagnosis for the patient based on the patient's manifestations is A. hyperthermia related to acute infectious process. B. chronic pain related to ineffective pain management. C. risk for injury related to disorientation and confusion. D. ineffective airway clearance related to retained secretions.

A. hyperthermia related to acute infectious process. The patient with pneumococcal pneumonia is acutely ill with fever and the systemic manifestations of fever, such as chills, thirst, headache, and malaise. Interventions that monitor temperature and aid in lowering body temperature are appropriate. Ineffective airway clearance would be manifested by adventitious breath sounds and difficulty producing secretions. Disorientation and confusion are not noted in this patient and are not typical unless the patient is very hypoxemic. Pleuritic pain is an acute pain that is due to inflammation of the pleura.

5. A client is diagnosed with a spontaneous pneumothorax necessitating the insertion of a chest tube. What is the best explanation for the nurse to provide this client? a. "The tube will drain fluid from your chest." b. "The tube will remove excess air from your chest." c. "The tube controls the amount of air that enters your chest." d. "The tube will seal the hole in your lung."

Answer B. The purpose of the chest tube is to create negative pressure and remove the air that has accumulated in the pleural space.

7. A client has a history of chronic obstructive pulmonary disease (COPD). As the nurse enters the client's room, his oxygen is running at 6 L/min, his color is flushed and his respirations are 8/min. What should the nurse do FIRST? a. Obtain a 12-lead EKG b. Place client in high Fowler's position c. Lower the oxygen rate d. Take baseline vital signs

Answer C. A low oxygen level acts as a stimulus for respiration. A high concentration of supplemental oxygen removes the hypoxic drive to breathe, leading to increased hypoventilation, respiratory decompensation, and the development of or worsening of respiratory acidosis. Unless corrected, it can lead to the client's death.

A climber attempts an assault on a high mountain in the Andes and reaches an altitude of 5000 meters (16,400 ft) above sea level. What will happen to his arterial PCO2 and pH? a) Both will be lower than normal. b) The pH will rise and PCO2 will fall. c) Both will be higher than normal due to the physical exertion. d) The pH will fall and PCO2 will rise

B The climber will suffer from a respiratory alkalosis. The decline in the PO2 with altitude will stimulate breathing to offset the hypoxia. Carbon dioxide is driven from the blood faster than it is produced in the tissues so PCO2 falls and pH rises.

40. Select all that apply. Atelectasis can be caused by A. long-term smoking. B. inadequate surfactant. C. localized airway obstruction. D. an increase in lung expansion. E. an increase in elastic recoil.

BCE The collapse of lung tissue has several causes, including reduced lung expansion, localized airway obstruction, inadequate surfactant, and an increase in elastic recoil. Smoking, although harmful, does not in itself cause atelectasis.

The nurse is caring for a patient with Tuberculosis, the nurse understands that second line medications include: Select all that apply: a. ethambutol HCL b. Isoniazid c. aminosalicyclate sodium d. capreomycin e. ethionamide

C, D, E a&b are first line drugs.

10. During discharge teaching for a 65-year-old patient with emphysema and pneumonia, which of the following vaccines should the nurse recommend the patient receive? A. S. aureus B. H. influenzae C. Pneumococcal D. Bacille Calmette-Guérin (BCG)

C. Pneumococcal The pneumococcal vaccine is important for patients with a history of heart or lung disease, recovering from a severe illness, age 65 or over, or living in a long-term care facility.

The RN is preparing to give a bed bath to an immobilized pt w/ TB. The RN should wear which item when performing this care? Surgical mask and gloves. Particulate respirator, gown, and gloves Particulate respirator and protective eyewear Surgical mask, gown, and protective eyewear

Particulate respirator, gown, and gloves.

The community health RN is conducting an educational session w/ community members regarding this symptoms associated w/ TB. Which is one of the first manifestations associated with TB? Dyspnea Chest pain A bloody, productive cough A cough with the expectoration of mucoid sputum

A cough with the expectoration of mucoid sputum

A client admitted with a gunshot wound to the abdomen is transferred to the intensive care unit after an exploratory laparotomy. I.V. fluid is being infused at 150 ml/hour. Which assessment finding suggests that the client is experiencing acute renal failure (ARF)? a. Urine output of 250 ml/24 hours b. Temperature of 100.2° F (37.8° C) c. Serum creatinine level of 1.2 mg/dl d. Blood urea nitrogen (BUN) level of 22 mg/dl

A ARF, characterized by abrupt loss of kidney function, commonly causes oliguria, which is characterized by a urine output of 250 ml/24 hours. A serum creatinine level of 1.2 mg/dl isn't diagnostic of ARF. A BUN level of 22 mg/dl or a temperature of 100.2° F (37.8° C) wouldn't result from this disorder.

A client is taking spironolactone (Aldactone) to control her hypertension. Her serum potassium level is 6 mEq/L. For this client, the nurse's priority should be to assess her: a. electrocardiogram (ECG) results. b. neuromuscular function. c. bowel sounds. d. respiratory rate

A Although changes in all these findings are seen in hyperkalemia, ECG results should take priority because changes can indicate potentially lethal arrhythmias such as ventricular fibrillation. It wouldn't be appropriate to assess the client's neuromuscular function, bowel sounds, or respiratory rate for effects of hyperkalemia

57. When admitting a patient with the diagnosis of asthma exacerbation, the nurse will assess for which of the following potential triggers? (Select all that apply.) A. Exercise B. Allergies C. Emotional stress D. Decreased humidity

A,B,C Although the exact mechanism of asthma is unknown, there are several triggers that may precipitate an attack. These include allergens, exercise, air pollutants, respiratory infections, drug and food additives, psychologic factors, and GERD.

52. When assessing a patient's sleep-rest pattern related to respiratory health, the nurse would ask if the patient: (Select all that apply.) A. Has trouble falling asleep B. Awakens abruptly during the night C. Sleeps more than 8 hours per night D. Has to sleep with the head elevated

A,B,D The patient with sleep apnea may have insomnia and/or abrupt awakenings. Patients with cardiovascular disease (e.g., heart failure that may affect respiratory health) may need to sleep with the head elevated on several pillows (orthopnea). Sleeping more than 8 hours per night is not indicative of impaired respiratory health.

56. When admitting a 45-year-old female with a diagnosis of pulmonary embolism, the nurse will assess the patient for which of the following risk factors? (Select all that apply.) A. Obesity B. Pneumonia C. Hypertension D. Cigarette smoking

A,C,D Research has demonstrated an increased risk of pulmonary embolism in women associated with obesity, heavy cigarette smoking, and hypertension. Other risk factors include immobilization, surgery within the last 3 months, stroke, history of DVT, and malignancy.

16. A client with emphysema should receive only 1 to 3 L/minute of oxygen, if needed, or he may lose his hypoxic drive. Which of the following statements is correct about hypoxic drive? a. The client doesn't notice he needs to breathe. b. The client breathes only when his oxygen levels climb above a certain point. c. The client breathes only when his oxygen levels dip below a certain point. d. The client breathes only when his carbon dioxide level dips below a certain point.

ANSWER C. Clients with emphysema breathe when their oxygen levels drop to a certain level; this is known as the hypoxic drive. They don't take a breath when their levels of carbon dioxide are higher than normal, as do those with healthy respiratory physiology. If too much oxygen is given, the client has little stimulus to take another breath. In the meantime, his carbon dioxide levels continue to climb, and the client will pass out, leading to a respiratory arrest.

The nurse is caring for a patient with COPD and pneumonia who has an order for arterial blood gases to be drawn. Which of the following is the minimum length of time the nurse should plan to hold pressure on the puncture site? A. 2 minutes B. 5 minutes C. 10 minutes D. 15 minutes

B. 5 minutes Following obtaining an arterial blood gas, the nurse should hold pressure on the puncture site for 5 minutes by the clock to be sure that bleeding has stopped. An artery is an elastic vessel under higher pressure than veins, and significant blood loss or hematoma formation could occur if the time is insufficient.

24. A nurse is preparing to establish oxygen therapy for a patient with COPD, and the physician's prescription reads "oxygen per nasal cannula at 5 L per minute." Which of the following actions should the nurse take? A. Administer the oxygen as prescribed. B. Call the physician and question the correct flow rate of the oxygen. C. Establish the oxygen as prescribed and obtain an ABG. D. Change the delivery device from a nasal cannula to a simple oxygen mask.

B. Call the physician and question the correct flow rate of the oxygen. The nurse should call the physician immediately and question the flow rate for delivery of the oxygen before implementation. Oxygen is used cautiously in patients with COPD because of longstanding hypoxemia serving as the respiratory drive mechanism. If high levels of oxygen are administered, the respiratory drive can be obliterated. Changing the device to a simple oxygen mask may alter the oxygen concentration being delivered to the patient and will further enhance the obliteration of the patient's respiratory drive. Obtaining an ABG sample is not a priority at this time, and the action does not address the validity of the prescribed oxygen dosing for the patient.

When caring for a patient who developed acute respiratory distress syndrome (ARDS) as a result of a urinary tract infection (UTI), the nurse is asked by the patient's family how a urinary tract infection could cause lung damage. Which response by the nurse is appropriate? a. "The infection spread through the circulation from the urinary tract to the lungs." b. "The urinary tract infection produced toxins that damaged the lungs." c. "The infection caused generalized inflammation that damaged the lungs." d. "The fever associated with the infection led to scar tissue formation in the lungs."

C Rationale: The pathophysiologic changes that occur in ARDS are thought to be caused by inflammatory and immune reactions that lead to changes at the alveolar-capillary membrane. ARDS is not directly caused by infection, toxins, or fever.

When prone positioning is used in the care of a patient with acute respiratory distress syndrome (ARDS), which information obtained by the nurse indicates that the positioning is effective? a. The skin on the patient's back is intact and without redness. b. Sputum and blood cultures show no growth after 24 hours. c. The patient's PaO2 is 90 mm Hg, and the SaO2 is 92%. d. Endotracheal suctioning results in minimal mucous return.

C Rationale: The purpose of prone positioning is to improve the patient's oxygenation as indicated by the PaO2 and SaO2. The other information will be collected but does not indicate whether prone positioning has been effective.

A client hospitalized for treatment of a pulmonary embolism develops respiratory alkalosis. Which clinical findings commonly accompany respiratory alkalosis? a. Nausea or vomiting b. Hallucinations or tinnitus c. Light-headedness or paresthesia d. Abdominal pain or diarrhea

C The client with respiratory alkalosis may complain of light-headedness or paresthesia (numbness and tingling in the arms and legs). Nausea, vomiting, abdominal pain, and diarrhea may accompany respiratory acidosis. Hallucinations and tinnitus rarely are associated with respiratory alkalosis or any other acid-base imbalance.

46. Nursing assessment findings of jugular vein distention and pedal edema would be indicative of which of the following complications of emphysema? A. Acute respiratory failure B. Pulmonary edema caused by left-sided heart failure C. Fluid volume excess secondary to cor pulmonale D. Secondary respiratory infection

C. Fluid volume excess secondary to cor pulmonale Cor pulmonale is a right-sided heart failure caused by resistance to right ventricular outflow due to lung disease. With failure of the right ventricle, the blood emptying into the right atrium and ventricle would be slowed, leading to jugular venous distention and pedal edema.

8. Which of the following clinical manifestations would the nurse expect to find during assessment of a patient admitted with pneumococcal pneumonia? A. Hyperresonance on percussion B. Fine crackles in all lobes on auscultation C. Increased vocal fremitus on palpation D. Vesicular breath sounds in all lobes

C. Increased vocal fremitus on palpation. A typical physical examination finding for a patient with pneumonia is increased vocal fremitus on palpation. Other signs of pulmonary consolidation include dullness to percussion, bronchial breath sounds, and crackles in the affected area.

31. The nurse determines that the patient understood medication instructions about the use of a spacer device when taking inhaled medications after hearing the patient state which of the following as the primary benefit? A. "Now I will not need to breathe in as deeply when taking the inhaler medications." B. "This device will make it so much easier and faster to take my inhaled medications." C. "I will pay less for medication because it will last longer." D. "More of the medication will get down into my lungs to help my breathing."

D. "More of the medication will get down into my lungs to help my breathing." A spacer assists more medication to reach the lungs, with less being deposited in the mouth and the back of the throat.

1. The arterial blood gas (ABG) readings that indicate compensated respiratory acidosis are a PaCO2 of A. 30 mm Hg and bicarbonate level of 24 mEq/L. B. 30 mm Hg and bicarbonate level of 30 mEq/L. C. 50 mm Hg and bicarbonate level of 20 mEq/L. D. 50 mm Hg and bicarbonate level of 30 mEq/L.

D. 50 mm Hg and bicarbonate level of 30 mEq/L. If compensation is present, carbon dioxide and bicarbonate are abnormal (or nearly so) in opposite directions (e.g., one is acidotic and the other alkalotic).

7. Which of the following physical assessment findings in a patient with pneumonia best supports the nursing diagnosis of ineffective airway clearance? A. Oxygen saturation of 85% B. Respiratory rate of 28 C. Presence of greenish sputum D. Basilar crackles

D. Basilar crackles The presence of adventitious breath sounds indicates that there is accumulation of secretions in the lower airways. This would be consistent with a nursing diagnosis of ineffective airway clearance because the patient is retaining secretions.

The nurse is instructing a patient about pursed lip breathing and the pt asks the nurse about its purpose. The nurse should tell the pt. that the primary purpose of pursed lip breathing is to: a. promote oxygen intake b. strengthen the diaphragm c. strengthen the intercostal muscles d. promote carbon dioxide elimination.

D. Pursed lip breathing facilitates maximal expiration for pt's . with obstructive lung disease

18. In the case of pulmonary embolus from deep vein thrombosis, which of the following actions should the nurse take first? A. Notify the physician. B. Administer a nitroglycerin tablet sublingually. C. Conduct a thorough assessment of the chest pain. D. Sit the patient up in bed as tolerated and apply oxygen.

D. Sit the patient up in bed as tolerated and apply oxygen.The patient's clinical picture is consistent with pulmonary embolus, and the first action the nurse takes should be to assist the patient. For this reason, the nurse should sit the patient up as tolerated and apply oxygen before notifying the physician.

12. After admitting a patient to the medical unit with a diagnosis of pneumonia, the nurse will verify that which of the following physician orders have been completed before administering a dose of cefotetan (Cefotan) to the patient? A. Serum laboratory studies ordered for AM B. Pulmonary function evaluation C. Orthostatic blood pressures D. Sputum culture and sensitivity

D. Sputum culture and sensitivityThe nurse should ensure that the sputum for culture and sensitivity was sent to the laboratory before administering the cefotetan. It is important that the organisms are correctly identified (by the culture) before their numbers are affected by the antibiotic; the test will also determine whether the proper antibiotic has been ordered (sensitivity testing). Although antibiotic administration should not be unduly delayed while waiting for the patient to expectorate sputum, all of the other options will not be affected by the administration of antibiotics.

25. The nurse identifies the nursing diagnosis of activity intolerance for a patient with asthma. The nurse assesses for which of the following etiologic factor for this nursing diagnosis in patients with asthma? A. Anxiety and restlessness B. Effects of medications C. Fear of suffocation D. Work of breathing

D. Work of breathingWhen the patient does not have sufficient gas exchange to engage in activity, the etiologic factor is often the work of breathing. When patients with asthma do not have effective respirations, they use all available energy to breathe and have little left over for purposeful activity.

15. A 69-year-old client appears thin and cachectic. He's short of breath at rest and his dyspnea increases with the slightest exertion. His breath sounds are diminished even with deep inspiration. These signs and symptoms fit which of the following conditions? a. ARDS b. Asthma c. Chronic obstructive bronchitis d. Emphysema

NSWER D. In emphysema, the wall integrity of the individual air sacs is damaged, reducing the surface area available for gas exchange. Very little air movement occurs in the lungs because of bronchiole collapse, as well. In ARDS, the client's condition is more acute and typically requires mechanical ventilation. In asthma and bronchitis, wheezing is prevalent.


Set pelajaran terkait

Stress Management and Prevention part 2

View Set

Writing and Balancing Chemical Equations

View Set

Principal liability to third parties for contract

View Set